Site Loader

Содержание

Закон Ома в интегральной форме

Для того, чтобы перейти к интегральной форме записи закона Ома для участка проводника, на котором действуют две силы, введем понятие линии тока.

Линия тока – кривая, в каждой точке которой вектор плотности тока направлен по касательной к этой кривой. В этом случае вектор плотности находится из соотношения:

где τ ⃗ – единичный вектор касательной к линии тока.

Предположим, что удельное сопротивление (r) и напряженность поля движущих сил (E ⃗) на поперечном сечении проводника однородны, т.к. E ⃗ однородна, то j ⃗ так же однородная величина. Возьмем произвольное значение поперечного сечения цепи – S. Тогда:

, а значит

Последнее равенство до множим на dl (элементарное перемещение вдоль вектора плотности тока):

где
  • dφ – элементарный сброс потенциала электростатического поля,
  • dε – элементарная работа сторонних сил по перемещению единичного положительного заряда (ЭДС).
Отсюда:

Учитывая, что ρ/S dl=dR (элементарное сопротивление), запишем закон Ома в интегральной форме:

Закон Ома в интегральной форме для неоднородного участка цепи

Проинтегрируем получившееся соотношение на конкретном участке цепи постоянного тока между поперечными сечениями S1 и S2:

интегральный закон Ома для участка цепи

где:

  • – сопротивление участка,
  • – работа сторонних сил на перемещении единичного положительного заряда по данному участку цепи ЭДС участка,
  • – работа электростатических сил на перемещении единичного положительного заряда по данному участку цепи (напряжение участка),
  • – абсолютная величина работы сил сопротивления на перемещении единичного положительного заряда по данному участку цепи (падение напряжения участка).

Запишем значение напряжения при постоянном токе:

Отсюда запишем закон Ома:

Таким образом закон Ома в интегральной форме – это закон изменения механической энергии единичного положительного заряда на этом участке. В арифметическом виде этот закон можно записать так:

Решение задач

Какой будет плотность тока в металлическом проводнике с удельным сопротивлением ρ постоянного сечения, имеющем длину l, если напряжение, которое приложено к проводу равно U?

Дано:Решение:
  • Плотность тока можно найти по формуле —
Пространство между пластинами плоского конденсатора заполняет неоднородное плохо проводящее вещество, удельная проводимость которого изменяется в соответствии с линейным законом: в направлении перпендикулярном пластинам. Известно, что расстояние между пластинами – d, площадь пластин конденсатора – S. Каким будет ток через этот конденсатор, если напряжение на нем станет равно U?
Дано:Решение:
  • d
  • S
  • U
  • Запишем закон Ома —
  • Отсюда можем найти силу тока —

  • Ответ

3.2.2. Закон Ома в интегральной и дифференциальной форме

Пусть по проводнику длиной l и сечением S течет ток I. В проводнике создается электрическое поле напряженности E, а 1 и 2 – потенциалы на концах проводника. В случае однородного проводника величину 1 2 = U можно назвать падением напряжения на участке проводника.

Закон Ома: сила тока, текущего по однородному участку проводника, прямо пропорциональна падению напряжения на проводнике:

— закон Ома в интегральной форме

где R – электрическое сопротивление проводника.

Размерность сопротивления в СИ: [R] = В/А = Ом.

Ом – сопротивление такого проводника, в котором при напряжении в 1 В течет ток 1А.

Сопротивление зависит от геометрических размеров и формы проводников, материала и температуры проводников. Для цилиндрического проводника

где — удельное сопротивление проводника.

Удельное сопротивление численно равно сопротивлению проводника длиной 1 м и площадью поперечного сечения 1 м2. Размерность удельного сопротивления в СИ: [] = Омм.

Величина, обратная сопротивлению, называется проводимостью.

Величина, обратная удельному сопротивлению, называется удельной проводимостью:

Единица, обратная Ом, называется Сименсом

[См].

Учитывая выше написанные уравнения, а также , получим:

– закон Ома в дифференциальной форме.

3.2.3. Сторонние силы. Закон Ома для цепи, содержащей эдс

Для возникновения и существования электрического тока необходимо:

  1. наличие свободных носителей тока – заряженных частиц, способных перемещаться упорядоченно;

  2. наличие электрического поля, энергия которого должна каким-то образом восполняться.

Соединим проводником два тела с зарядами +q и q. Кулоновские силы заставляют электроны перемещаться по проводнику. Возникнет ток. Однако тела при этом будут разряжаться, разность потенциалов уменьшится, ток быстро прекратится.

Т.е. если в цепи действуют только силы электростатического поля, то происходит перемещение носителей таким образом, что потенциалы всех точек цепи выравниваются и электростатическое поле исчезает.

Следовательно, поле кулоновских сил не может являться причиной постоянного электрического тока.

Ток в проводнике нейтрализует заряды на его концах. Для поддержания постоянного тока необходимо поддерживать постоянную разность потенциалов, следовательно, разделять заряды. Электрические силы разделять заряды не могут.

Силы, разделяющие заряды, имеют неэлектрическую природу и называются сторонними силами.

Устройство, в котором действуют сторонние силы, называется источником тока.

Сторонние силы заставляют заряды двигаться внутри источника тока против сил поля. Благодаря этому в цепи поддерживается постоянная разность потенциалов.

Перемещая заряды, сторонние силы совершают работу за счет энергии, затраченной в источнике тока. Например, в электрофорной машине разделение зарядов происходит за счет механической работы, в гальваническом элементе – за счет энергии химических реакций и т.д.

Величина, равная работе сторонних сил по перемещению единичного положительного заряда, называется электродвижущей силой (ЭДС).

Обозначим — вектор напряженности поля сторонних сил.

Результирующее поле, действующее на заряды в проводнике, в общем случае

Плотность тока в цепи

.

– закон Ома в дифференциальной форме для цепи, содержащей ЭДС.

Рассмотрим участок AB замкнутой цепи, содержащей ЭДС (рис.3.18). Выделим мысленно малый элемент dl.

Плотность тока на этом участке опишется уравнением . Умножим скалярно обе части этого равенства наи проинтегрируем по участкуAB:

Рассмотрим каждый интеграл в отдельности:

а)

где АВ – разность потенциалов между точками A и B.

Разность потенциалов численно равна работе кулоновских сил по перемещению единичного положительного заряда из т.A в т.B;

б)

где — ЭДС.

ЭДС, действующая на участке цепи, численно равна работе сторонних сил по перемещению единичного положительного заряда из т.

A в т.B;

в)

где RAB – сопротивление участка AB.

С учетом выше сказанного можно получить:

— закон Ома для участка цепи с ЭДС.

Частные случаи:

  1. если на данном участке цепи источник тока отсутствует, то получаем закон Ома для однородного участка цепи:

  1. если цепь замкнута (=0), то получим закон Ома для замкнутой цепи:

где — ЭДС, действующая в цепи, R – суммарное сопротивление всей цепи, rвнутр – внутреннее сопротивление источника тока, Rвнеш – сопротивление внешней цепи;

  1. если цепь разомкнута, то I = 0 и 12 = 21, т.е. ЭДС, действующая в разомкнутой цепи равна разности потенциалов на ее концах.

  2. В случае короткого замыкания сопротивление внешней цепи Rвнеш = 0 и сила тока в этом случае ограничивается только величиной внутреннего сопротивления источника тока.

Величина IRAB = UAB называется падением напряжения на участке AB.

Падение напряжения на участке AB численно равно работе кулоновских и сторонних сил по перемещению единичного положительного заряда из т.A в т.B.

Если цепь замкнута, то 1 = 2 и

– закон Ома для замкнутой цепи.

Если участок цепи не содержит ЭДС, то

Закон ома в интегральной форме для однородного участка цепи — Блог Маланьина



В предыдущем параграфе мы выяснили, что в однородном проводнике средняя. Она выражает в дифференциальной форме закон Ома для неоднородного участка цепи. Закон Ома в интегральной форме для однородного участка цепи не содержащего ЭДС Для однородного линейного проводника выразим R через? ? Выражение 4 или 5 представляет собой закон Ома для неоднородного участка цепи в интегральной форме. Закон Ома для однородного участка цепи в интегральной форме. Закон ома в интегральной форме для однородного участка цепи. Закон Ома для однородного участка цепи и закон Ома в дифференциальной форме. Здесь предполагается, что проводник однородный и имеет постоянную площадь. Закон ома в интегральной форме для неоднородного и однородного участка цепи. Закон Ома интегральной форме для однородного участка цепи. Однородный и неоднородный участки цепи. Эту формулу еще называют законом Ома в интегральной форме. Закон Ома в дифференциальной форме Закон Ома в интегральной форме для однородного участка цепи не содержащего ЭДС Для однородного линейного. Закон ома для однородного участка цепи в интегральной и дифференциальной форме. С учетом этого закон Ома для замкнутой цепи дифференциальной форме будет иметь вид В таком. От закона Ома дифференциальной форме легко перейти к интегральной форме. Закон Ома для неоднородного участка цепи в интегральной форме является. В своей работе Ом записал закон в. Закон Ома в интегральной форме для однородного участка цепи не содержащего ЭДС Для однородного линейного проводника выразим R через, 7. Ом экспериментально установил, что сила тока в. Рассмотрим в однородной изотропной среде элементарный объем с. Ома в дифференциальной форме. 4 приходим к закону Ома для однородного. Закон Ома для однородного участка цепи. Далее учтем, что величина алгебраическая если, то 0 если же, то. 22 есть закон Ома в интегральной обобщенной форме для цепи. Используя закон Ома для однородного участка цепи, формулу 2. Cпециально для Вас подготовлен образовательный документ Закон Ома для однородного участка цепи в интегральной и дифференциальной форме. Ома для неоднородного участка цепи в интегральной форме, который являетсяобобщенным законом Ома. Закон Ома для участка цепи 8 класс. Имеет смысл освежить эти знания, в статье мы напомним трактовку закона, составленного Омом, для однородного и неоднородного участка и полной цепи. Закон Ома в интегральной форме для однородного участка цепи не содержащего ЭДС. Закон Ома в интегральной форме. 23, можно записать закон Ома в интегральной форме для участка. Ом экспериментально установил закон, согласно которому сила тока. Закон Ома для тока в газахЗакон Ома, представляющий один из важнейших законов. E однородна, то так же однородная величина. Ома для однородного участка. Последнее уравнение выражает собой закон Ома в интегральной форме для участка цепи. Закон Ома в дифференциальной форме Закон Ома в интегральной форме для однородного участка цепи не содержащего.Последнее уравнение выражает собой закон Ома в интегральной форме для участка цепи, содержащего эдс и формулируется следующим образом падение.

Закон Ома для неоднородного участка цепи

| на главную | доп. материалы | физика как наука и предмет | электричество и электромагнетизм |

Организационные, контрольно-распорядительные и инженерно-технические услуги
в сфере жилой, коммерческой и иной недвижимости. Московский регион. Официально.

Мы рассматривали закон Ома (см. (98.1)) для однородного участка цепи, т. е. такого, в котором не девствует э.д.с. (не действуют сторонние силы). Теперь рассмотрим неоднородный участок цепи, где действующую э.д.с. на участке 12 обозначим через а приложенную на концах участка разность потенциалов — через j1—j2.

Если ток проходит по неподвижным проводникам, образующим участок 1—2, то работа А12 всех сил (сторонних и электростатических), совершаемая над носителями тока, по закону сохранения и превращения энергии равна теплоте, выделяющейся на участке. Работа сил, совершаемая при перемещении заряда Q0 на участке 12, согласно (97.4),

                                          (100.1)

Э.д.с. как и сила тока I, величина скалярная. Ее необходимо брать либо с положительным, либо с отрицательным знаком в зависимости от знака работы, совершаемой сторонними силами. Если э.д.с. способствует движению положительных зарядов в выбранном направлении (в направлении 1—2), то > 0. Если э.д.с. препятствует движению положительных зарядов в данном направлении, то < 0. За время t в проводнике выделяется теплота (см. (99.5))

                                     (100.2)

Из формул (100.1) и (100.2) получим

                                                              (100.3)

откуда

                                        (100.4)

Выражение (100.3) или (100.4) представляет собой закон Ома для неоднородного участка цепи в интегральной форме, который является обобщенным законом Ома.

Если на данном участке цепи источник тока отсутствует (=0), то из (100.4) приходим к закону Ома для однородного участка цепи (98.1):

(при отсутствии сторонних сил напряжение на концах участка равно разности потенци­алов (см. § 97)). Если же электрическая цепь замкнута, то выбранные точки 1 и 2 со­впадают, j1=j2; тогда из (100.4) получаем закон Ома для замкнутой цепи:

где  — э.д.с., действующая в цепи, R — суммарное сопротивление всей цепи. В общем случае R=r+R1, где r — внутреннее сопротивление источника тока, R1со­противление внешней цепи. Поэтому закон Ома для замкнутой цепи будет иметь вид

Если цепь разомкнута и, следовательно, в ней ток отсутствует (I = 0), то из закона Ома (100.4) получим, что =j1—j2, т. е. э.д.с., действующая в разомкнутой цепи, равна разности потенциалов на ее концах. Следовательно, для того чтобы найти э.д.с. источника тока, надо измерить разность потенциалов на его клеммах при разомкнутой цепи.


Закон Ома в интегральной и дифференциальной форме. — КиберПедия

Закон Ома в интегральной форме:

— для участка цепи: «Сила тока на однородном участке электрической цепи прямо пропорциональна напряжению на данном участке и обратно пропорциональна сопротивлению этого участка »:

— для всей цепи:

где – электродвижущая сила, В;

– сопротивление всех элементов цепи, Ом;

внутреннее сопротивление источника питания, Ом;

сила тока, А.

Закон Ома в дифференциальной форме:

— для участка цепи: «Плотность тока в каждой точке однородного участка цепи пропорциональна напряженности электрического поля в этой же точке»:

Где – вектор плотности тока, А/м²;

– удельная проводимость, См= ;

– вектор напряженности электрического поля, В/м.

Работа и мощность электрического тока. Закон Джоуля-Ленца в интегральной и дифференциальной форме.

За время t по участку электрической цепи будет перенесён заряд и при этом будет совершена работа:

где – электрический заряд, Кл;

– напряжение, В;

– сила тока, А;

– время, с.

Работа, совершаемая в единицу времени – мощность электрического тока:

Закон Джоуля-Ленца в дифференциальной форме: «Мощность тепла , выделяемого в единице объёма среды при протекании постоянного электрического тока, равна произведению плотности электрического тока на величину напряженности электрического поля »:

где – удельная электрическая проводимость, См= .

Закон Джоуля-Ленца в интегральной форме:

где – полное количество теплоты, выделенное за промежуток времени от до , Дж;

– сила тока, А;

– сопротивление. Ом.

Закон Ома для участка цепи и для замкнутой цепи, содержащей э.д.с.

Закон Ома для участка цепи:

«Сила тока на однородном участке электрической цепи прямо пропорциональна напряжению на данном участке и обратно пропорциональна сопротивлению этого участка »:

Закон Ома для неоднородного участка цепи (содержащего ЭДС):

где – разность потенциалов (напряжение), В;

– электродвижущая сила, В;

– сопротивление участка, Ом.

Если направление тока совпадает с направлением обхода, его принято считать положительным; если источник тока повышает потенциал в направлении обхода (источник посылает ток в направлении обхода), то ЭДС такого источника считается положительной.

Закон Ома для замкнутой цепи, содержащей ЭДС:

где – электродвижущая сила, В;

– сопротивление всех элементов цепи, Ом;

внутреннее сопротивление источника питания, Ом;

сила тока, А.

 

Законы Кирхгофа.

Первое правило Кирхгофа: «Алгебраическая сумма токов, сходящихся в узле, равна нулю:

».

Второе правило Кирхгофа: «В любом замкнутом контуре алгебраическая сумма напряжений равна алгебраической сумме ЭДС, встречающихся в этом контуре:

где – число источников ЭДС;

– число ветвей в замкнутом контуре;

ток и сопротивление -той ветви, А, Ом».

Правило знаков:

1) ЭДС положительна, если ее направление совпадает с направлением произвольно выбранного обхода контура;

2) падение напряжения на резисторе положительно, если направление тока в нем совпадает с направлением обхода.

 

О законе Ома: интегральная и дифференциальная формы

Немецкий физик Георг Ом в XIX веке экспериментально вывел основную закономерность, по которой функционируют и проектируются электрические цепи. Она заключается в том, что сила тока прямо пропорциональна напряжению и обратно пропорциональна сопротивлению. Данная формулировка описывает закон Ома в интегральной форме.

Дифференциальный вид закона Ома

Действие электродвижущих сил

Работая с электричеством, следует помнить, что сопротивление проводника зависит от его размера, формы и материала, из которого он изготовлен. Поэтому данный показатель при решении теоретических и прикладных задач рассчитывается как отношение длины к площади, умноженное на величину удельного сопротивления материала.

Дополнительная информация. Величина сопротивления также зависит и от температуры, где находится проводник. Как правило, такая зависимость носит линейный характер: чем выше температура, тем больше сопротивление проводящего вещества.

Чтобы в цепи появилось электричество, необходимо наличие в ней свободных заряженных частиц (обычно электронов). Кроме того, они должны обладать способностью перемещаться в определенном направлении (от источника тока к его потребителю, от отрицательно заряженного предмета к месту с положительным зарядом). Такое движение создает электрическое поле. Следовательно, чтобы движение частиц не прекращалось, необходимо энергию этого поля каким-либо образом восполнять.

Если соединить проводом два тела, у одного из которых положительный, а  у другого отрицательный заряд, из-за кулоновских сил начнется движение электронов. Однако достаточно быстро такое перемещение прекратится, поскольку разница потенциалов из-за действия законов природы восстановится. Таким образом, наличие в электрической цепи только электростатических сил явно недостаточно, чтобы обеспечить постоянное движение электронов в сети.

Важно! Чтобы поддерживать постоянное наличие тока в сети, необходимо обеспечивать разность потенциалов на ее концах. Естественным образом такую ситуацию создать невозможно. Следовательно, необходимы сторонние силы, которые называются электродвижущими.

Электродвижущие силы

Благодаря внешнему воздействию, электроны движутся в направлении, противоположном действию электрического поля, за счет чего поддерживается постоянная разница потенциалов. Создает электродвижущие силы источник тока за счет механического действия, химической реакции и так далее. Следовательно, интегрального вида записи закона Ома для описания функционирования электрической цепи недостаточно, поскольку, помимо разницы потенциалов и сопротивления проводника, на движение электронов действует еще ряд факторов. Решить эту задачу позволяет закон Ома в дифференциальной форме.

Движение тока по неоднородным проводникам

Дифференциальная форма записи выявленной Омом закономерности особенно актуальна, когда проводящий элемент по своему составу неоднороден – на всем протяжении движения электронов у него меняется площадь сечения и уровень сопротивления. Это создает определенные сложности при расчете мощности источника тока, параметров изоляции и так далее, чтобы обеспечить стабильность работы сети, а главное – ее безопасность.

Чтобы сформулировать закон Ома в дифференциальной форме, следует представить проводник не как однородное тело, а как набор бесконечного числа бесконечно малых частей. Это позволит считать каждый элемент однородным, а значит, у него постоянная толщина и постоянный уровень сопротивления, и на таком бесконечно малом участке действуют стандартные принципы закона Ома.

Обратите внимание! При записи закономерности Ома в дифференциальном виде необходимо ввести такие понятия, как плотность тока и удельная проводимость, поскольку именно они являются ключевыми для расчета параметров электрической цепи с неоднородным проводником.

Под плотностью понимается векторная величина, которая демонстрирует уровень силы тока, протекающий через единицу площади. Учет плотности приводит к тому, что при неоднородном проводящем элементе в схеме  потребуется установка различных дополнительных устройств для выравнивания напряжения и обеспечения стабильности и безопасности работы.

Плотность и проводимость проводника

Удельная проводимость – это величина, обратная удельному сопротивлению, которая позволяет оценить способность единицы какого-либо вещества обеспечивать прохождение через себя электронов. Знание такой характеристики также позволяет корректно спроектировать схему из различных проводников.

Итак, закон Ома является базовым для понимания устройства электрической цепи. Возможность записать его в разных видах позволяет учесть при проектировании схем устройств и приборов толщину, проводимость и другие характеристики материалов. Необходимо отметить, что такая деятельность требует знаний в области высшей математики (владение основами дифференциальных и интегральных вычислений).

Видео

Оцените статью:

Закон Ома в интегральной форме

Для однородного участка цепи, то есть для участка, на котором не действуют сторонние силы, закон Ома записывается в форме (2.8). Рассмотрим теперь неоднородный участок цепи 1-2 (рис. 2.8), где действует ЭДС источника и на концах которого приложена разность потенциалов .

 

P AAAA//8DAFBLAwQUAAYACAAAACEAl4xLNeAAAAALAQAADwAAAGRycy9kb3ducmV2LnhtbEyPwU7D MAyG70i8Q2QkLmhLKKwaXdMJTdoRNFYkrmnrpdUap2qyrfD0mBO72fKn39+fryfXizOOofOk4XGu QCDVvunIavgst7MliBANNab3hBq+McC6uL3JTdb4C33geR+t4BAKmdHQxjhkUoa6RWfC3A9IfDv4 0ZnI62hlM5oLh7teJkql0pmO+ENrBty0WB/3J6fBHn/s5q2Mbhuncrcj+fD1VL1rfX83va5ARJzi Pwx/+qwOBTtV/kRNEL2GRC25S9QwW6Q8MPGSpAsQFaPqWYEscnndofgFAAD//wMAUEsBAi0AFAAG AAgAAAAhALaDOJL+AAAA4QEAABMAAAAAAAAAAAAAAAAAAAAAAFtDb250ZW50X1R5cGVzXS54bWxQ SwECLQAUAAYACAAAACEAOP0h/9YAAACUAQAACwAAAAAAAAAAAAAAAAAvAQAAX3JlbHMvLnJlbHNQ SwECLQAUAAYACAAAACEAyqttX88IAAAUcAAADgAAAAAAAAAAAAAAAAAuAgAAZHJzL2Uyb0RvYy54 bWxQSwECLQAUAAYACAAAACEAl4xLNeAAAAALAQAADwAAAAAAAAAAAAAAAAApCwAAZHJzL2Rvd25y ZXYueG1sUEsFBgAAAAAEAAQA8wAAADYMAAAAAA== «>

 

На рассматриваемом участке работа всех приложенных сил (сторонних и электростатических), совершаемая над носителями тока, согласно (2.6) равна:

В этой формуле ЭДС берется либо с положительным, либо с отрицательным знаком. Если ЭДС способствует движению положительных
зарядов в направлении обхода (в направлении 1-2), то есть внутри источника обход совпадает с перемещением зарядов от катода к аноду, то
(рис. 2.8, а). Если ЭДС препятствует движению положительных зарядов
в направлении обхода, то (рис. 2.8, б).

По закону сохранения и превращения энергии работа равна теплоте, выделяющейся на участке 1-2 за время t (эта теплота определяется согласно закону Джоуля-Ленца):

(2.20)

 

Приравнивая (2.6) и (2.20), получим

 

(2.21)

или

(2.22)

 

где R – суммарное сопротивление, включающее в себя внутреннее сопротивление r источника тока и сопротивление внешней цепи.

Выражение (2.21) или (2.22) есть закон Ома в интегральной (обобщенной) форме для цепи постоянного тока.

Действительно, если на данном участке цепи источник тока отсутствует ( ), то из (2.22) приходим к закону Ома для однородного участка цепи:

Если электрическая цепь замкнута (точки 1 и 2 совпадают), то . Тогда из (2.22) получаем закон Ома для замкнутой цепи:

Наконец, если цепь разомкнута, то и из (2.22) получаем, что , следовательно, для экспериментального определения ЭДС
источника тока необходимо измерить разность потенциалов на его зажимах при разомкнутой нагрузке (режим холостого хода цепи).


Узнать еще:

Что такое неоднородный участок цепи. Закон Ома неоднородный участок цепи

Электродвижущая сила.

Если в проводнике создается электрическое поле и не принимаются меры для его поддержания, то очень быстрое перемещение носителей тока приведет к исчезновению поля внутри проводника и прекращению тока. Чтобы поддерживать ток в течение длительного времени, необходимо непрерывно снимать положительные заряды, принесенные сюда током с конца проводника с более низким потенциалом j 2, и переносить их на конец с высоким потенциалом (рис.56.1).

Создаваемое в проводнике электрическое поле не может осуществить такую ​​передачу заряда. Для существования постоянного тока необходимо действие некоторых других сил (не кулоновских сил), которые перемещают заряды против электрических сил и поддерживают постоянство электрических полей. Это могут быть магнитные силы, заряды могут разделяться за счет химических реакций, диффузии носителей заряда в неоднородной среде и т. Д. Чтобы подчеркнуть разницу между этими силами и силами кулоновского взаимодействия, их принято обозначать термином внешние силы .Устройства, в которых движение свободных зарядов под действием внешних сил называется источниками тока. Сюда входят электромагнитные генераторы, термоэлектрические генераторы, солнечные батареи. Отдельную группу составляют химические источники тока: гальванические элементы, батареи и топливные элементы.

Действие внешних сил можно охарактеризовать, введя понятие напряженности поля внешних сил :.

Работа сторонних сил по перемещению заряда q на участке дл можно выразить следующим образом:

по всей длине участка л :

.(56,1)

Величина, равная отношению работы внешних сил по перемещению заряда к этому заряду, называется электродвижущей силой (ЭДС):

. (56,2)

В проводнике, по которому течет ток, напряженность электрического поля является суммой напряженности поля кулоновских сил и внешних сил:

Тогда для плотности тока мы можем написать

Замените векторы их проекциями в направлении замкнутого контура и умножьте обе части уравнения на dl :

После подстановки, результирующее уравнение сокращается до

Проинтегрируем полученное выражение по длине электрической цепи:

Интеграл в левой части уравнения — это сопротивление R на участке 1-2.В правой части уравнения значение первого интеграла численно равно работе кулоновских сил по перемещению единичного заряда из точки 1 в точку 2 — это разность потенциалов. Значение второго интеграла численно равно работе внешних сил по перемещению единичного заряда из точки 2 в точку 1 — это электродвижущая сила. В соответствии с этим сводим уравнение (56.3) к виду

Значение IR , равное произведению силы тока и сопротивления участка цепи, называется падение напряжения на цепи.Падение напряжения численно равна работе, совершаемой при перемещении единичного заряда под действием внешних сил и сил электрического поля (кулоновского).

Часть схемы, содержащая ЭДС, называется гетерогенной частью. Сила тока в таком сечении находится по формуле (56.4):

Учитывая, что источник тока можно подключить к цепи двумя способами, заменяем знак перед ЭДС на «±»:

Выражение (56.5) — это закон Ома для неоднородного участка цепи. Знаки «+» или «-» учитывают, как внешние силы влияют на течение тока в указанном направлении: они способствуют или препятствуют (рис. 56.2).

Если участок цепи не содержит ЭДС, т.е. однороден, то из формулы (56.5) следует

Из формулы (56.5) следует

где IR — падение напряжения на внешней части цепи, Ir — падение напряжения на внутренней части цепи.

Следовательно, ЭДС источника тока равна сумме падений напряжения на внешней и внутренней частях цепи .

Дифференциальная форма закона Ома . Найдите связь между плотностью тока j и напряженностью поля E в одной и той же точке проводника. В изотропном проводнике упорядоченное движение носителей тока происходит в направлении вектора E . Следовательно, направления векторов j и E совпадают. Рассмотрим в однородной изотропной среде элементарный объем с образующими, параллельными вектору E длиной, ограниченным двумя эквипотенциальными участками 1 и 2 (рис.4.3).

Обозначим их потенциалы и, а среднюю площадь поперечного сечения через. Используя закон Ома, получаем для тока, или для плотности тока, поэтому

Перейдем к пределу при, тогда рассматриваемый объем можно считать цилиндрическим, а поле внутри него однородным, так что

где E — напряженность электрического поля внутри проводника. Учитывая, что j и E совпадают по направлению, получаем

.

Это соотношение представляет собой дифференциальную форму закона Ома для однородного участка цепи . Величина называется проводимостью. Помимо электростатических сил, на носители тока в неоднородном участке цепи действуют и внешние силы, поэтому плотность тока в этих участках пропорциональна сумме интенсивностей. Учет этого приводит к дифференциальной форме закона Ома для неоднородного участка цепи .

.

При прохождении электрического тока по замкнутой цепи на свободные заряды действуют силы стационарного электрического поля и внешние силы.Причем в определенных частях этой цепи ток генерируется только стационарным электрическим полем. Такие участки цепочки называются однородными . В некоторых частях этой цепи, помимо сил стационарного электрического поля, действуют и внешние силы. Часть цепи, на которую действуют внешние силы, называется участком неоднородной цепи .

Чтобы узнать, от чего зависит сила тока в этих секциях, необходимо уточнить понятие напряжения.

Рассмотрим сначала однородный участок цепи (рис. 1, а). В этом случае только силы стационарного электрического поля совершают работу по перемещению заряда, и этот участок характеризуется разностью потенциалов Δ φ . Разность потенциалов на концах графика Δ φ = φ 1− φ 2 = AKq , где A K — работа сил стационарного электрического поля. Неоднородный участок контура (рис.1, б), в отличие от однородного участка, содержит источник ЭДС, и на этом участке к работе сил электростатического поля добавляется работа внешних сил. По определению Aelq = φ 1− φ 2, где q — положительный заряд, перемещающийся между любыми двумя точками цепи; φ 1− φ 2 — разность потенциалов точек начала и конца рассматриваемого участка; Astq = ε .Тогда поговорим о стрессе за напряжением: E stats. е. п. = Е э / стат. п. + E сторона Напряжение U на участке цепи — физическая скалярная величина, равная суммарной работе внешних сил и сил электростатического поля при перемещении одиночного положительного заряда в этом участке:

U = AKq + Astorq = φ 1− φ 2+ ε .

Эта формула показывает, что в общем случае напряжение в данном участке цепи равно алгебраической сумме разности потенциалов и ЭДС в этом участке.Если на площадке действуют только электрические силы ( ε = 0), то U = φ 1− φ 2. Таким образом, только для однородного участка цепи понятия напряжения и разности потенциалов совпадают.

Закон Ома для неоднородного участка цепи имеет вид:

I = Ur = φ 1− φ 2+ εR ,

где R — полное сопротивление неоднородной области.

ЭДС ε может быть как положительным, так и отрицательным. Это связано с полярностью включения ЭДС в секции: если направление, создаваемое источником тока, совпадает с направлением тока, протекающего в секции (направление тока в секции совпадает внутри источника с направлением от отрицательный к положительному полюсу), т.е. ЭДС способствует перемещению положительных зарядов в этом направлении, тогда ε > 0, иначе, если ЭДС препятствует движению положительных зарядов в этом направлении, то ε

Электродвижущая сила.

Закон Ома для замкнутого контура и неоднородного участка цепи.

Закон Ома для замкнутой цепи означает, что. Величина тока в замкнутой цепи, которая состоит из источника тока с внутренним сопротивлением, а также внешнего сопротивления нагрузки. Он будет равен отношению электродвижущей силы источника к сумме внешнего и внутреннего сопротивлений.

Закон Ома для неоднородного участка цепи

При прохождении электрического тока по замкнутой цепи на свободные заряды действуют силы стационарного электрического поля и внешние силы.Причем в определенных частях этой цепи ток генерируется только стационарным электрическим полем. Такие участки цепи называются однородными. В некоторых частях этой цепи, помимо сил стационарного электрического поля, действуют и внешние силы. Участок цепи, на который действуют внешние силы, называется неоднородным участком цепи.

Чтобы узнать, от чего зависит сила тока в этих секциях, необходимо уточнить понятие напряжения.



Фиг.1

Рассмотрим сначала однородный участок цепи (рис. 1, а). В этом случае только силы стационарного электрического поля совершают работу по перемещению заряда, и этот участок характеризуется разностью потенциалов Δφ. Разность потенциалов на концах графика, где АК — работа сил стационарного электрического поля. Неоднородный участок цепи (рис. 1, б), в отличие от однородного участка, содержит источник ЭДС, и на этом участке к работе сил электростатического поля добавляется работа внешних сил.По определению, где q — положительный заряд, который перемещается между любыми двумя точками цепи; — разность потенциалов точек начала и конца рассматриваемого участка; . Затем они говорят о стрессе за напряжением: Приблиз. е. п. = Ee / stat. п. + Estor. Напряжение U на участке цепи — это физическая скалярная величина, равная полной работе внешних сил и сил электростатического поля при перемещении одиночного положительного заряда в этом участке:

Эта формула показывает, что в общем случае напряжение в данном участке цепи равно алгебраической сумме разности потенциалов и ЭДС в этом участке.Если на узел действуют только электрические силы (ε = 0), то. Таким образом, только для однородного участка схемы понятия напряжения и разности потенциалов совпадают.

Закон Ома для неоднородного участка цепи имеет вид:

где R — полное сопротивление неоднородного участка.

ЭДС ε может быть как положительной, так и отрицательной. Это связано с полярностью включения ЭДС в секции: если направление, создаваемое источником тока, совпадает с направлением тока, протекающего в секции (направление тока в секции совпадает внутри источника с направлением от отрицательный к положительному полюсу), т.е. ЭДС способствует движению положительных зарядов в этом направлении, тогда ε> 0, в противном случае, если ЭДС препятствует перемещению положительных зарядов в этом направлении, то ε
Правила кихгоф.

Работа и текущая мощность. Эффект термического тока. Закон Джоуля-Ленца.

Когда ток течет через однородный участок цепи, электрическое поле выполняет свою работу. За время Δ t по цепи протекает заряд Δ q = I Δ t.Электрическое поле на выбранной станции делает свою работу

Мощность электрического тока равна отношению текущей работы Δ A к временному интервалу Δ t, в течение которого эта работа была выполнена:

Работа электрического тока в СИ выражается в джоулях (Дж), мощность — в ваттах (Вт).

Рассмотрим теперь полную цепь постоянного тока, состоящую из источника с электродвижущей силой и внутренним сопротивлением r и внешнего однородного участка с сопротивлением R.Закон Ома для полной цепи записывается как

Первое слагаемое в левой части Δ Q = RI 2 Δ t — это тепло, выделяющееся на внешней части контура за время Δ t, второе слагаемое Δ Q ist = r I 2 Δ t — это теплота тепло, выделяемое внутри источника за то же время.

Выражение I Δ t равно работе внешних сил Δ A st, действующих внутри источника.

Когда электрический ток течет по замкнутой цепи, работа внешних сил Δ A st преобразуется в тепло, выделяющееся во внешней цепи (Δ Q) и внутри источника (Δ Q источник).

Δ Q + Δ Q источник = Δ A артикул = I Δ t
.

Следует отметить, что электрическое поле не входит в это соотношение. Когда ток течет по замкнутой цепи, электрическое поле не работает; поэтому тепло генерируется только внешними силами, действующими внутри источника. Роль электрического поля сводится к перераспределению тепла между различными частями цепи.

Внешняя цепь может быть не только проводником с сопротивлением R, но и каким-нибудь устройством, потребляющим электроэнергию, например, двигателем постоянного тока. В этом случае под R нужно понимать эквивалентное сопротивление нагрузки. Энергия, выделяемая во внешней цепи, может быть частично или полностью преобразована не только в тепло, но и в другие формы энергии, например, в механическую работу, выполняемую электродвигателем. Поэтому вопрос использования энергии источника тока имеет большое практическое значение.

Наиболее часто используемым в электротехнике соотношением между основными электрическими величинами является закон Ома, установленный эмпирическим путем в 1826 году немецким физиком Георгом Омом. С его помощью устанавливается связь между напряжением (электродвижущей силой) и сопротивлением элементы этой цепи, и сила проходящего тока.

Электрические параметры, которые описываются законом Ома:

  • Сила тока определяется количеством заряда, проходящего через проводник за некоторое время, обозначается буквой I, единица измерения — ампер (А ).Входит в базовые единицы международной системы C;
  • Напряжение электрическое, единица измерения — вольты, понятие ввел все тот же Георг Ом. Вольт можно выразить через работу по движению заряда, выделяемая мощность при токе 1 ампер, имеет эталонные источники в виде высокостабильных гальванических ячеек. Часто обозначается как разность потенциалов, в некоторых случаях используется понятие электродвижущей силы (ЭДС). Для обозначения можно использовать буквы U, V;
  • R — сопротивление (электрическое), указывает на свойства проводника, препятствующие прохождению тока.Существенно зависит от материала проводника и температуры. Единица измерения — 1 Ом, обозначение — Ом или Ом.

Классическая формулировка закона Ома: сила тока в цепи прямо пропорциональна напряжению и обратно пропорциональна сопротивлению.

Это выражение действительно для электрической цепи, которая не содержит дополнительной электродвижущей силы, обеспечивающей электрический ток, цепи, определяемой как однородная. В большинстве случаев используется такая формула.На практике часто бывает необходимо рассчитать величину тока, протекающего через какой-либо элемент с известным сопротивлением, для этого достаточно измерить падение напряжения (разность потенциалов) на выводах этого устройства, например, резистора. Для любых двух заданных значений можно вычислить неизвестное, и помимо значений, включенных в выражение, определяется электрическая мощность.

Важно! При расчете используются значения только одной размерности — целые значения вольт, ампер, ом или соответствующие им кратные и дробные единицы.

Неоднородная цепочка

Закон Ома для одного участка цепи не учитывает наличие источника питания, его свойства не учитываются в расчетах. Для схемы, называемой гетерогенной, содержащей ЭДС любого вида и ее источник, в известной формуле, следует добавить внутреннее сопротивление самого устройства питания:

Здесь E — ЭДС источника напряжения, r — его внутреннее сопротивление. Варианты названий — закон Ома для неоднородного участка цепи, для полной или замкнутой цепи.Выражение мало отличается от приведенного выше — вместо напряжения здесь ЭДС и сопротивление источника питания.

Следует отметить, что понятие внутреннего сопротивления имеет смысл исключительно для химических источников тока, в случае использования других устройств, таких как любые источники питания без батарей, они говорят о выходном сопротивлении и нагрузочной способности этот блок.

В практических приложениях закон Ома для неоднородного участка цепи в такой форме используется редко, в основном для измерения внутреннего сопротивления батареи и других батарей.

Закон также применяется к переменному напряжению, если сопротивление является активной нагрузкой. С его помощью определяются эффективные (среднеквадратичные) параметры цепи. В случае индуктивной, емкостной или сложной нагрузки и для разных частот сопротивление является реактивным, значительно отличающимся от того, которое измеряется обычным методом — омметром.

Закон Ома был получен практическим путем, поэтому он не может быть фундаментальным, но он точно описывает взаимосвязь между наиболее часто используемыми электрическими величинами.

Видео

В этой статье мы поговорим о законе Ома, формулах для полной цепи (замкнутой), участка цепи, неоднородного участка цепи, в дифференциальной и интегральной форме, переменного тока, а также для магнитопровода. Вы узнаете, какие материалы соответствуют и не соответствуют закону Ома, а также где это происходит.
: постоянный ток, протекающий по проводнику, прямо пропорционален напряжению, приложенному к его концам, и обратно пропорционален сопротивлению.

Закон Ома был сформулирован немецким физиком и математиком Георгом Омом в 1825-26 годах на основе опыта. Это экспериментальный закон, а не универсальный — он применим к определенным материалам и условиям.

Закон Ома является частным случаем более позднего и более общего — второго закона Кирхгофа

Ниже будет представлен видеоролик, объясняющий закон Ома на пальцах.

Формула закона Ома для цепочки Раздел

Сила постоянного тока, протекающего по проводнику, пропорциональна напряжению, приложенному к его концам.В Интернете эту формулу часто называют первым законом Ома:

U- напряжение

I — сила (сила) тока

R — Сопротивление

Электрическое сопротивление:

Коэффициент пропорциональности R называется электрическим сопротивлением или сопротивлением.

Отношение напряжения к току для данного проводника постоянно:

Единица электрического сопротивления 1 Ом (1 Ом):

Сопротивление резистора равно 1, если приложенное напряжение составляет 1 вольт, а ток — 1 ампер.

Зависимость электрического сопротивления от размера направляющей:

Сопротивление токопроводящего участка с постоянным поперечным сечением R прямо пропорционально длине этого участка li, обратно пропорционально площади поперечного сечения S:

R- электрическое сопротивление

ρ — удельное сопротивление

I- длина направляющей

S- площадь поперечного сечения

Это соотношение было подтверждено экспериментально британским физиком Хамфри Ди в 1822 году до разработки закона Ома.

Закон Ома для замкнутой (полной) цепи

— это величина силы (интенсивности) тока в цепи тока, которая зависит от сопротивления нагрузки и от источника тока (Е), его еще называют вторым законом Ома.

Лампочка — это потребитель источника тока, соединяя их вместе, они образуют законченную электрическую цепь. На картинке выше вы можете увидеть полную электрическую схему, состоящую из батареи и лампы накаливания.

Электричество проходит через лампу накаливания и через саму батарею. Следовательно, ток, проходящий через лампу, впоследствии будет проходить через батарею, то есть сопротивление лампы добавляется к сопротивлению батареи.

Сопротивление нагрузки (лампочка), называется внешнее сопротивление , а сопротивление источника тока (батареи) внутреннее сопротивление . Сопротивление батареи обозначается латинской буквой r.

Когда электричество течет по цепи, внутреннее сопротивление самого элемента сопротивляется протеканию тока, и поэтому тепловая энергия теряется в самом элементе.

  • E = электродвижущая сила в вольтах, В
  • I = ток в амперах, А
  • R = сопротивление нагрузки цепи в Ом, Ом
  • r = внутреннее сопротивление ячейки в Ом, Ом

Мы можем изменить это уравнение;

В этом уравнении появляется ( В, ), то есть конечная разность потенциалов , измеренная в вольтах (В).Это разность потенциалов на выводах ячейки при протекании тока в цепи, она всегда меньше, чем у ячеек ЭДС.

Закон Ома для неоднородного участка цепи

Если только потенциальные силы ( Рисунок 1a ), то закон Ома записывается в известной форме. Если в круге также проявляется действие внешних сил ( Рисунок 2b ), то закон Ома принимает вид откуда. Это закон Ома для любой части цепи. .

Закон Ома можно распространить на весь круг.Соединив точки 2 и 1 ( Рисунок 3c ), преобразуем разность потенциалов в ноль, и с учетом сопротивления источника тока закон Ома принимает вид. Это выражение закона Ома для полной цепи. .

Последнее выражение может быть представлено в различных формах. Как известно, напряжение на внешней секции зависит от нагрузки, т.е.
или, или.

В этих выражениях Ir — это падение напряжения внутри источника тока, и также видно, что напряжение U меньше ε на Ir .Причем, чем больше внешнее сопротивление по сравнению с внутренним, тем больше U приближается к ε.

Рассмотрим два особых случая, касающихся внешнего сопротивления цепи.

1) R = 0 — это явление называется коротким замыканием. Тогда из закона Ома имеем -, то есть ток в цепи увеличивается до максимума, а падение внешнего напряжения U 0. При этом в источнике выделяется большая мощность, что может привести к его неисправности.

2) R = ∞ , то есть электрическая цепь разорвана, значит, a. Итак, в данном случае ЭДС численно равна напряжению на выводах открытого источника тока.

Закон Ома в дифференциальной форме

Закон Ома можно представить так, что он не связан с размером проводника. Выбрать участок проводника Δ л , , на концах которого приложены потенциалы φ 1 и φ 2.При средней площади поперечного сечения проводника Δ S и плотности тока Дж то ток

Если Δ l → 0, затем взяв предел отношений,. Итак, мы наконец получили, или в векторной форме, это выражение закона Ома в дифференциальной форме . Этот закон выражает силу тока в произвольной точке проводника в зависимости от его свойств и электрического состояния.

Закон Ома для переменного тока

Это уравнение представляет собой обозначение закона Ома для цепей переменного тока относительно их значений амплитуды.Понятно, что справедливо будет для действующих значений силы и тока:.

Для цепей переменного тока возможен случай, когда, то есть U L = У С . Поскольку эти напряжения находятся в противофазе, они компенсируют друг друга. Такие условия называются резонансом напряжения . Резонанса можно добиться либо при ω = const , изменяя ОТ и L , либо при постоянном ОТ и L поднять ω, что называется резонансным .Как видно — .

Характеристики резонанса напряжения следующие:

Наконец, из (2) — (4) имеем выражение для закона Ома в интегральной форме

, который он установил экспериментально.

Толкование закона Ома

Сила тока, которая является действием приложенного напряжения, ведет себя пропорционально его напряжению. Например: если приложенное напряжение удваивается, это также удваивает силу тока (силу тока).

Помните, что закону Ома удовлетворяет только часть материалов — в основном металлы и керамические материалы.

Когда действует закон Ома и какие материалы соответствуют и не соответствуют закону Ома

Закон Ома — это экспериментальный закон, выполняемый для некоторых материалов (например, металлов) для фиксированных условий тока, в частности температуры проводника.

Материалы, относящиеся к закону Ома, называются омическими проводниками или линейными проводниками. Примерами проводников, соответствующих закону Ома, являются металлы (например, медь, золото, железо), некоторые керамические изделия и электролиты.

Материалы, не относящиеся к закону Ома, в которых сопротивление является функцией силы тока, протекающего через них, называются нелинейными проводниками. Примеры закона Ома — полупроводники и газы.

Закон Ома не выполняется при изменении параметров проводника, особенно температуры.

Закон

Ома для односторонних неоднородных ланцюг. Закон Ома для односторонней ставки. Определитель Провайдера. Способы хранения КОР в процессе имплантации электромагнетизма

Как только в фонарях на носу бренчания остается только напряженность электростатического поля, то происходит смещение носов (передается зловоние позитива) от точек с более высоким потенциалом к ​​точкам с более низким потенциалом.Процесс доведен до уровня потенциальности во всех точках ланцета и вплоть до развития электрического поля. Следовательно, чтобы реализовать постоянный бренчание, необходимо иметь пристройку в ланцеюзе, проем в здании и увеличивать потенциал для развития роботизированных сил неэлектростатической ходьбы. Такие насадки называются джерэлами струму … принудительно неэлектростатическая ходьба , Д_ют на зарядке со стороны джерела зоба, называется снаружи .

Природа внешних сил может быть храброй. Например, в гальванических элементах обнаруживается запах энергии химических реакций между электродами и электролитами; в генераторе — для механизма механической энергии, обертывания ротора генератора и т.д. системы. Со стороны схлопнувшегося поля внешних сил электрический заряд схлопывается в середине барабана против сил электростатического поля.

Джерело зоб и счастливый зоб (опир R ) Положите закрытое копье (рисунок 6.3).

Джерело зоб можно охарактеризовать опорой r (Определение внутренней части ланцюга) и электрической силой (EPC) ε — не поможет роботу внешних сил переместить единичный положительный заряд в одну подвеску с отрицательного полюса на положительный

Изображение джерела струму на схемах показано на рис. 6.3, б.

Следует отметить, что увидеть его в электрическом копье по дому не всегда возможно, в любом случае.Вне силы могут работать все улан. Например, EPC Inductions Vinikє рисует во всех точках контур, который находится в переменном магнитном поле.

Дилянка ланцюга, снимающая и кулоновидная силы сразу, называется неоднородной дилянкой ланцюга (рис. 6.4). Робот кулоновских сил, сдвигая электрический заряд на центральную ступень, характеризует увеличение потенциала (), а робот сторонних сил — на центральную ступень фонаря EPC ε 1,2:

(6.7)


Для гетерогенного кола вводится новое значение, которое называется sprung U 1,2 , Вон характеризует нестандартного робота сторонних и кулоновских сил на неоднородной копье. кувшин со сменой разового положительного заряда:

(6,8)

На односторонней ланцюге (), подъездная дорога разницы потенциал:

Для описания силового воздействия на силу в поле внешних сил заряда (по аналогии с электростатическим полем) введите силовую характеристику — напряженность поля внешних сил:

Тоди формул (6.7) и (6.8) могут быть представлены в средстве просмотра:

(6,9)

для EPC ε, Пользуюсь замкнутой системой пенсий, с виразом (6.9) можно

tobto EPC ε направление циркуляции вектора напряжения внешних сил по полному замкнутому контуру. Следует отметить, что поле внешних сил с точки зрения электростатического поля не является потенциальным.

Закон Ома для односторонней ставки:

Дилянка ланцюга называется годовалой, так как зоб джерело не поступает на склад.I = U / R, 1 Ом — опир такого проводника, в котором сила тока 1А при 1В.

Величина поддержки зависит от формы и мощности материала направляющей. Для одностороннего цилиндрического проводника R = ρl / S, ρ — это значение, которое может быть выделено из порочного материала — конкретное определение речи, с ρ = RS / l далее, что (ρ) = 1 Ом * м. . γ = 1 / ρ.

Экспериментально установлено, что при регулировке температуры электрическая работа металлов увеличивается.Даже не при более низких температурах скорость роста металлов составляет ~ абсолютная температура p = α * p 0 * T, p 0 — скорость роста при 0 ° C, α — температурный коэффициент. Для более крупных металлов α = 1/273 = 0,004 К -1. p = p 0 * (1+ α * t), t — температура в оС

Согласно классической электронной теории металлов в металлах с электроникой с идеальной кристаллической решеткой, носитель не разрушается (p = 0).

Причина появления электрической опоры — боковые дефекты и физические дефекты кристаллической решетки, а также термическая деградация атомов.Амплитуда числа выпавших атомов из t. Появление подставки для домашних животных из-за функции складывания:

p (T) = p остаток + p id. , P ost — излишек ходатайства opir, p id. — Идеально подходит для металла.

Идеально для образования абсолютно чистого металла, и это не начинается с использования тепловых всплесков атомов. В павильоне окраинного миркувана уд. opir id. металл виновен в прагматичности до 0 при T → 0. Однако определенное описание функции хранения сумм независимых доданков, что в связи с появлением домов и в.дефекты кристаллической решетки опорной структуры при уменьшении t → до сингулярного роста штифта. p отдых. В некоторых случаях температура невысокая и проходит через минимум. Ценность остального. Лучше всего опору закладывать при появлении дефектов в растворе и на месте домов.

j = γ * E — закон Ома в дифференцированной форме, описывающий процесс в точке кожи провайдера, de j — сила звона, E — напряженность электрического поля.

Ланцюг включает в себя резистор R і джерело зоба. При неоднородной конструкции копья на носовой части барабана существует предел электростатическим силам внешней силы. Вне власти построения порядка носа в бренчании, например электростатическом. На неоднородной копье поля к полю электрических зарядов добавляется поле внешних сил, которое задается приборной панелью EPC. Закон Ома в дифференцированной форме: j = γE. Наглядная формула для типа неоднородной направляющей j = γ (E + E *) (1).

Согласно закону Ома в дифференцированной форме для неравномерной ставки, можно перейти к интегральной форме закона Ома для цепочки долей. У всех видна неоднородная дилянка. В более новой модели может быть неприемлемо поперечное перекрытие направляющей. Признается, что середина дома ланцюга — это простая линия, так как зоб мы будем называть контуром, так счастлив:

1. В кожном покрове сетчатки значения j, γ, E, E * могут иметь одинаковые значения перпендикулярно контуру.

2. j, E и E * в точках кожи выравниваются по одной линии с контуром.

Viberemo довольно прямолинеен по контуру. Не поворачивайте прямо для изменения с 1 на 2. При этом элемент проводника площадью S и элемент контура dl. Спроецированный вектор, входящий в (1) на элемент контура dl: j = γ (E + E *) (2).

Первый контур дорожной проекции зоба на площадку: I = jS (3).

Электропроводность Pitoma: γ = 1 / ρ.Заминючи в (2) I / S = 1 / ρ (E + E *). Умножьте на dl и проинтегрируйте перемычку ∫Iρdl / S = ∫Eedl + ∫E * edl. Vrahuєmo, ρdl / S = R и ∫Eedl = (φ 1 -φ 2), ∫E * edl = ε 12, IR = ε 12+ (φ 1 -φ 2). ε 12, jak і I — величина алгебраическая, было бы неприлично, если отрицательный полюс создавался по прямой 1-2, в обратном направлении, ε 12> 0, то положительный. Якшо ع перешкою ​​до ручки положительных носов, по прямой, то ε 12

Из последней формулы I = (φ 1 -φ 2) + (-) ε 12 / R.Формула qia превращает закон Ома в неравномерную ставку. Когда вы выходите из этого, вы можете отклонить закон Ома за неравномерную ставку. Между прочим, ε 12 = 0, также I = (φ 1 -φ 2) / R, I = U / R, а также закон Ома для закрытой ставки: φ 1 = φ 2, то есть I = ع / R, de R — сумарный оп_р всего ланцеха: I = ع / R 0 + r.

На практике видно, что для создания устойчивого потока в замкнутой фурме необходимо или принципиально природа, а не кулон, що д_ют на заряд, за виньеткой сил электрического опора (Кулонівское силы)).Такая дилянка называется неоднородной дилянкой ланцюга. На нижней части наведения, приклад такой дилянки.

Напряженность поля в прямой точке стрельчатой ​​дороги векторная сумма поля кулоновских сил и поля внешних сил:

Закон Ома может быть сформулирован для неравномерного стержня кола — сила удара прямо пропорциональна нагрузке на центральную опору и выражена пропорционально ее основной опоре:

— формула закона Ома для неравномерной ставки.
  • I — сила зоба,
  • U12 — разлив на дилянци,
  • R — ланцюг новый опір.

Разность потенциалов робота характеризует электрическое поле путем передачи единственного положительного заряда (q) из точки 1 в точку 2:

— де φ1 и φ 2 — потенциалы на концах дилянки.

EPC характеризует робота внешних сил передачей единичного положительного заряда из точки 1 в точку 2: — де ε12 — EPC, численно дорогостоящие роботы путем смещения единичного положительного заряда контура.

Напряжение на стойке дилянци є сумма к роботу ЭП и внешних сил:


Закон Тоди Ома уже в поле зрения:

EPC может быть положительным или отрицательным. Це лежал в полярности включенного ЭПК в дилянку. Если весь зоб среднего джерела обошел с отрицательного полюса на положительный, то ЭПК положительный (див. Малюнок). Посторонние всегда старались относиться к роботу положительно. Если вы перейдете от положительного полюса к отрицательному, то EPC будет отрицательным.Кажется проще, если ЭПК с числом положительных зарядов, то ε> 0,


Решение задач по закону Ому для гетерогенной ставки

Вишначити бренчание, которое показано на изображении на маленькой дилянци А.В. EPC джерела 20 В, внутренний опир 1 Ом, потенциалы точек А и Б 15 В и 5 В, проводник опир 3 Ом.

Дилянка ланцюга, в любом случае, сторонних сил нет, но производить до ЕПК (рис. 1), называется односторонней.

Закон Ома для односторонней дилянки колы-бува установления экспериментально в 1826 г. Г. Ом.

Это хорошо с законом, сила удара I в однолинейном металлическом проводе прямо пропорциональна нагрузке U на концах проводника и намотана пропорционально опоре проводника R :

На блоке 2 изображена схема электрического колышка, позволяющая экспериментально пересмотреть закон. На дилянку МН ланцюга бызі входят направляющие, которые володют с небольшими опорами.

Напряжение на концах проводника измеряется вольтметром и может быть изменено с помощью дополнительного потенциометра. Сила зоба измеряется амперметром, который очень мал ( R A ≈ 0). График наложения силы зоба в проводнике от источника к новому — вольт-амперная характеристика проводника — направлен на малую 3. Кут вольт-амперной характеристики закладывает электрическую опора кондуктора R (Abo yogo electric G ):.

Сила удара заключается в закрытом ланцете, который накапливается от зоба от внутренней опоры и опоры от опоры, переноса размера EPC к сумме внутренней опоры от опоры и опоры. поддержка от саппорта.

8. OPIR І ПРОВИДЕННЫЙ провайдер. ИСТОРИЯ ПОДДЕРЖКИ ПОСТАВЩИКОВ ВИД ФИЗИЧЕСКОГО УМА

Когда электрический столб заклинивает, на беговой дорожке возникает разница потенциалов, выигрывает электрический бренчание.Электроэнергия поля заменяется с помощью проводника. В моей собственной России у нас есть возможность использовать атомы проводника и снабжать их собственной кинетической энергией.

Обладая таким рангом, электрони, проходя через проводника, создает опир собственноручно. Пропуская электрический звук через проводник, он перестанет нагреваться.

Электрическая поддержка поставщика (также известная как латинская буква r) используется для обобщения явления преобразования электрической энергии в тепло, когда электрическая система передается поставщиком.На схемах электрическая работа обозначена, как показано на рис. 18.

За одну опору принятия 1 Ом … Ом часто называют большой ореховой буквой Ω (омега). Для того, чтобы вместо того, чтобы писать: «Описание провайдера — 15 Ом», можно просто написать: r = 15 Ом.

1000 Ом называется от 1 до 1 (1 кОм или 1 к Ом).

1000000 Ом называется 1 м о м (1 мг или 1 МОм).

Недавно и параллельно с в электротехнике — есть два основных метода изготовления элементов электрического кола.В последний раз все элементы увязаны один в один, так что в него вошла дилянка ланцуга немыслимого вуза. При параллельном подключении все входы в локальные сети соединены двумя университетами, а с другими университетами нет никаких связей, а также чтобы не выходить за рамки разумного.

С последней мудростью гидов сила бренчания в наших гидах такая же.

При параллельном соединении пружины между двумя косяками используются для объединения элементов ланцюга, однако для всех элементов оно одинаковое.При таком же размере это звенящая опора для лансера, большая сумма ценностей, звенящая опора параллельно включению направляющих.

Як вначити загальный опир кола, Как в него все включено в последнюю очередь, но не в последнюю очередь? Використовуючи позитення, у шпоры У на беговой дорожке джерела зобом доривню суми капля напруга на дилянки ланцюга, можно написать:

U = U1 + U2 + U3

U1 = IR1 U2 = IR2 і U3 = IR3

ИК = ИК1 + ИК2 + ИК3

Винши в правой части ривности I для лука, можно принять IR = I (R1 + R2 + R3).

Добавив теперь оскорбительные части равенства к I, оставим остаток R = R1 + R2 + R3

В таком ранге мы подошли к висновке, но при последнем установлении опоры всей стрельчатой ​​дороги, сумме опоры рамы.

Электрический бренчание — в порядке некомпенсированного электрического заряда. Как только рухнет выигрыш у провайдера, то электрическое бренчание называется бренчанием провинций. Электрическое бренчание может быть победным с кулонами.Поле цих-сил называется кулоновским и характеризует напряженность Е-холода.

Заряды руха могут быть вызваны неэлектрическими силами, которые называются посторонними (магнитными, химическими). Э арт — напряжение поля сил цич.

Порядок электрических зарядов может происходить без каких-либо значительных сил (диффузия, химические реакции в струне). Для духа мира мы введем эффективное внешнее искусство E во все представление.

Повна робота по смещенному заряду на диалекте ланцюга:

Разділимо оскорбить часть оставшегося платежа на размер начисления, измененного в соответствии с начислением.

.

Ризница потенциал для дилянци ланцюга.

С другой стороны, стоимость колы называется значением, которое важно для всего робота, поскольку оно переходит, когда заряд смещается на центральное количество, к значению заряда. Tobto ВНЕШНИЙ ВИД НА ДИЛЯНЦИ КОЛА — ЧЕХИ ПОЛНОГО РОБОТА С УДАЛЕНИЕМ ДИЛЬЯНЦИ ОДНОГО ПОЛОЖИТЕЛЬНОГО ЗАРЯДА.

EPC, исходя из изменения, представляет собой стоимость, которая является дорогостоящей для роботов, которая может использоваться в качестве неэлектрических источников питания при смещении заряда до значения заряда. EPC — ЦЕПЬ РОБОТА ТРЕТЬИХ СИЛ Z ПЕРСОНАЛИЗИРОВАННЫЙ одиночный положительный ЗАРЯД НА участке ЦЕПИ.

Внешняя мощность в электрических уланах, как правило, в джерелях зоба. Если на ланцюге дилянци є джерело зоба, то такую ​​дилянку называют неоднородной.

Напряжения на неоднородном дизайне полос дороги к сумме развития потенциалов у детей дома дома и EPC перешли в новый.Когда вы получаете много EPC, вы попадаете в положительный, как если бы это был прямой зоб, он был напрямую связан с внешними силами, так что от минуса джерела к плюсу.

Пока у нас есть dilentzi nemaє dzherel struma, то в целом и только в общих чертах будет много дорогостоящего потенциала роста.

В закрытом ланцете для кожной перевязки, который можно использовать для создания замкнутого контура, можно написать:

Колебания потенциала гребешка и острия равны, значит.

Отже, (2),

тобто сума выпадают напруг в замкнутой цепи, составляя электрическую ставку на сумму EPC.

Роздилимо обиделась часть села (1) за обеденный дом.

De — напряжение суммированного поля, — напряжение бокового поля, — напряжение кулоновского поля.

Для односторонней дилянки колы.

Сила зоба, отсюда же — закон Ома в дифференциальной форме. Сила зоба в односторонней диленке колы прямо пропорциональна НАПРЯЖЕНИЮ ЭЛЕКТРОСТАТИЧЕСКОГО ПОЛЯ В ДИРЕКТОРЕ.

Якщо в дату lanceuga diane кулоновского и бокового поля (неоднородная диленция лантюги), тогда степень бренчания будет пропорциональна общему напряжению поля:

… То есть ,.

Закон Ома для смешанной ставки: СИЛА СТРУМА В НЕОДНОРОДНОМ ДИЛЯНЦИ КОЛА ПРЯМО пропорциональна целому ДИЛЯНЦИ И ОБРАТНО пропорционально его поддержке.

Если прямые E c t и E холодный старт, то EPC и потенциальное увеличение могут быть одного знака.

В закрытом ланцеусе V = 0 кулоновское поле консервативно.

Звидси:,

де Р — опир последней части ланцюга, р — опир внутренней части ланцюга (тобто джерел струму).

Закон Ома для закрытой ставки: СИЛА СТРУМА В закрытом ланцете прямо пропорциональна EPC ДЖЕРЕЛ Я завернут в пропорциональный овный опир-кола.

Правила Кирхгофа.

При разработке распределения электрических копий следуйте правилам Киркхгофа.

Точка Ланцюг, в которой три и более направляющих называются вузл. Это связано с законом экономии заряда суммы бренчания, которая поступает в университет и уходит с дороги на ноль. … (Перше правило Кирхгофа). Алгебраическая сумма зобов, проходящая через ВУЗОЛ, равна нулю.

Strum: при поступлении в университет участие положительно, а при поступлении в университет — отрицательно. Прямые в ланцюг-дылянках довольно сильно вибрируют.

Заработок (2) viplyaє, scho В течение часа хожу по алгебре BUD-ONE CLOSED CIRCUIT , — (другое правило Кирхгофа).

Обход контура изрядно вибрирует. Нагрузка на дилянцы кола положительная, так как бренчанию на дилянци несложно вовлечься в прямой обход контура. EPC задействуется положительно, так как при движении по контуру он переходит от отрицательного полюса к положительному.

Пока копейщики мстят m университетам, то по первому правилу возможно m-1 ривняння. Кожне нове ривняння может включать в себя один новый элемент. Снаружи количество ривнянов, сложенных по правилам Кирхгофа, увеличивается на количество диланок между школами, тобто. с номером бренчания.

Как записан закон Ома для замкнутой цепи. Закон Ома простым языком

Если точки 1 и 2 совпадают, то выражение закона Ома для узла принимает более простой вид:

где — полное сопротивление замкнутой цепи, включая внутреннее сопротивление источников, а — алгебраическая сумма ЭДС.в этой цепочке.

Ток, который возникает, когда внешнее сопротивление равно нулю, называется током короткого замыкания.

Лекция 10.

Подключение проводов.

Используя закон Ома для участка цепи, можно показать, что сопротивления последовательного и параллельного соединения проводников равны соответственно:

Проба:

Обратите внимание, что при параллельном соединении проводов общее сопротивление всегда меньше наименьшего сопротивления в параллельном соединении.Посмотреть на себя.

Закон Джоуля-Ленца.

Когда ток проходит через проводник, за счет сопротивления выделяется тепло, которое рассеивается в окружающей среде. Найдем это количество тепла. Воспользуемся для этого законом сохранения энергии и законом Ома.

Рассмотрим однородный участок цепи, на котором поддерживается постоянная разность потенциалов. В этом случае электрическое поле делает работу:

Если на объекте не происходит преобразования в механическую, химическую или другие виды энергии помимо тепловой энергии, то количество выделяемого тепла равно работе электрического поля:

.

В данном случае тепловая мощность равна:

Окончательное количество тепла находится путем интегрирования во времени:

Эта формула выражает закон Джоуля-Ленца. Механизм тепловыделения связан с преобразованием дополнительной кинетической энергии, приобретаемой носителями тока в электрическом поле, в энергию возбуждения колебаний решетки при столкновении носителей с атомами в узлах решетки.

Найдем выражение для закона Джоуля — Ленца в локальной форме.Для этого выделим в проводнике элементарный объем в виде цилиндра с образующей по вектору. Пусть сечение цилиндра равно, а его длина. Тогда, согласно закону Джоуля-Ленца, в этом объеме со временем выделяется количество тепла:

где — объем цилиндра. Разделив последнее соотношение на, получаем формулу, определяющую тепловую мощность, выделяемую на единицу объема проводника:

Удельная тепловая мощность измеряется в.

Полученное соотношение выражает закон Джоуля-Ленца в локальной форме: удельная тепловая мощность тока пропорциональна квадрату плотности тока и удельного сопротивления проводника в данной точке.

В этой форме закон Джоуля-Ленца применим к неоднородным проводникам любой формы и не зависит от природы внешних сил. Если на носители действуют только электрические силы, то по закону Ома:

Если участок цепи содержит источник ЭДС, то на носители тока будут действовать не только электрические, но и внешние силы.В этом случае тепло, выделяющееся в этой области, равно алгебраической сумме работы электрических и внешних сил.

Умножим закон Ома в интегральной форме на силу тока:

Здесь слева стоит (тепловая мощность), а справа алгебраическая сумма мощностей электрических и внешних сил, которая называется мощностью тока.

В замкнутой цепи:

тех. мощность тепловыделения равна силе внешних сил.

Дифференциальный закон Ома

V
Отделяем от массива проводника (по которому течет электрический ток I ) небольшой цилиндр, расположенный вдоль линий электрического тока в проводнике Рис.5.2. Пусть длина цилиндра дл и участка dS … Тогда

O
здесь

И
Используя определение плотности тока (5.1) и проводимости проводника (5.4) окончательно получаем выражение, которое называется дифференциальным законом Ома

Работа и мощность, производимые электрическим током

Когда заряд перемещается между точками с определенной разностью потенциалов, соответствующей падению напряжения U работа и произведенная мощность:

NS
Этот закон был получен экспериментально и получил название закона Джоуля-Ленца. Если, как и в предыдущем случае, перейти к рассмотрению малых объемов, то легко получить закон Джоуля — Ленца в дифференциальной форме (5.6-5,8):

Законы Кирхгофа

Первое правило Кирхгофа

Рассмотрим электрическую схему с разветвлением Рис.5.3. Точки ветвления будем называть узлами. При установившемся процессе, когда электрический ток, протекающий по цепи, постоянен, потенциалы всех точек в цепи также не изменяются. Это может произойти, если электрические заряды не накапливаются и не исчезают в узлах цепи.

Таким образом, в устойчивом состоянии количество электричества, поступающего в узел, равно количеству электричества, которое покинуло узел.Отсюда следует первое правило Кирхгофа:

Алгебраическая сумма сил сходящихся в узле электрических токов равна нулю (5.9) (токи, входящие в узел, принимаются со знаком +, а токи, выходящие из узла, со знаком -)

I1 + i2 + i3-i4-i5 = 0

ΣI i = 0 5,9.

Соединения проводов

На практике часто необходимо использовать разные соединения проводов.

NS Последовательное соединение Рис. 5.4.

NS
При таком подключении электрический ток на всех участках цепи и на всех ее элементах одинаковый I = I 1 = I 2 = I 3 =… I n … Напряжение на концах цепи между точками A и B представляет собой сумму напряжений на каждом из ее элементов U AB = U 1 + U 2 + U 3 +… U n … Таким образом.

Параллельное соединение Рисунок 5.5

Закон Ома для замкнутого контура, содержащего э.д.с.

R Рассмотрим неразветвленную электрическую цепь, содержащую E.D.S. ( E ) с внутренним сопротивлением r и содержащим внешнее сопротивление R Рисунок 5.6

Полная работа по перемещению заряда по всей цепи будет состоять из работы во внешней цепи и работы внутри источника А = А наружу + А источника .

Кроме того, работа во внешней цепи, связанная с величиной заряда, по определению является разностью потенциалов на внешней цепи (падение напряжения на внешней цепи) A снаружи / q = У . И работа по всей цепочке, относящейся к обвинению, согласно определению E.D.S. А / q = E … Отсюда E = U + A источник / q … С другой стороны A источник = I 2 РТ . Отсюда A Источник / q = Ir . Таким образом, окончательно получаем: E = U + Ir

или E = I ( R + r ) 5.12

Под E означает сумму всех ЭЦП. включены в неразветвленную цепь, а r и R означают сумму всех внутренних и внешних сопротивлений в неразветвленной цепи.

Сила тока одинакова для всей неразветвленной замкнутой цепи, содержащей EDC. прямо пропорциональна E.D.S. и обратно пропорционален импедансу цепи.

Второе правило Кирхгофа

Рассмотрим разветвленную схему Рисунок 5.7. Участок между двумя соседними узлами называется ветвью.Поскольку разветвление происходит только в соседних узлах, то внутри ветви сохраняется сила тока по величине и направлению. Любую цепочку можно рассматривать как набор цепей, и для каждой цепи верно:

В любом замкнутом контуре, мысленно изолированном от электрической цепи, алгебраическая сумма произведений сопротивлений соответствующих участков цепи, включая внутренние сопротивления источников и ток в цепи, равна алгебраической сумме всех ЭЦП.в цепочке

Закон Ома для замкнутой цепи

Если в проводнике создается электрическое поле и не принимаются меры по его поддержанию, то движение зарядов очень быстро приведет к тому, что поле внутри проводника исчезнет и ток прекратится, следовательно, для поддержания постоянный ток в течение длительного времени необходимо соблюдение двух условий: электрическая цепь должна быть замкнута; в электрической цепи, вместе с участками, в которых плюс

Если заряды движутся в направлении уменьшения потенциала, должны быть участки, в которых эти заряды движутся в направлении увеличения потенциала, то есть против сил электростатического поля (см. Часть схемы, показанную пунктирной линией). на рис.5).

Только силы неэлектростатического происхождения, называемые внешними силами, могут перемещать положительные заряды против сил электростатического поля. Величина, равная работе внешних сил по перемещению одиночного положительного заряда, называется электродвижущей силой (ЭДС) e , действующей в цепи или на ее участке. ЭДС и измеряется в вольтах (В). Источник ЭДС имеет некоторое внутреннее сопротивление в зависимости от его конструкции. Это сопротивление оказывается включенным последовательно с источником в общей электрической цепи.В качестве источников ЭМП используются электрохимические ячейки и генераторы постоянного тока (рис. 6).

Если неразветвленная замкнутая электрическая цепь (рис. 7) содержит несколько последовательно соединенных элементов с сопротивлением и источники ЭДС от e до , имеющие внутреннее сопротивление, то ее можно заменить эквивалентной схемой, показанной на рис. 6. Ток в Схема замещения определяется по закону Ома для замкнутой цепи:

;

ЭДС, как и сила тока, является алгебраической величиной.Если ЭДС способствует движению положительных зарядов в выбранном направлении, то e > 0, если ЭДС препятствует перемещению положительных зарядов в заданном направлении, то e e> 0, если мы войдем внутрь источника из положительный полюс к отрицательному, затем e

Рис. 6 Рис.7

Из закона Ома для замкнутой цепи следует, что падение напряжения U на выводах источника меньше ЭДС. Реально e , или e … Так как по закону Ома для однородного участка цепи напряжение на выводах истока, то

3) с помощью закона Ома для замкнутой цепи установить связь между силой тока и ЭДС.

Закон быстрого Ома

Закон Ома — это физический закон, который определяет соотношение между напряжением, током и сопротивлением проводника в электрической цепи. Назван в честь первооткрывателя Георга Ома.
Так получилось, что в этом разделе страницы были две словесные формулировки закона Ома:
1.Суть закона проста: если во время прохождения тока напряжение и свойства проводника не изменяются, то
ток в проводнике прямо пропорционален напряжению между концами проводника и обратно пропорционален сопротивлению проводника.
2. Закон Ома формулируется следующим образом: ток в однородном участке цепи прямо пропорционален напряжению, приложенному к участку, и обратно пропорционален характеристике участка, которая называется электрическим сопротивлением этого участка.
Также следует иметь в виду, что закон Ома является фундаментальным (основным) и может применяться к любой физической системе, в которой потоки частиц или полей действуют для преодоления сопротивления. Его можно использовать для расчета гидравлических, пневматических, магнитных, электрических, световых, тепловых потоков и т. Д., А также правил Кирхгофа, однако такое применение этого закона используется крайне редко в рамках узкоспециализированных расчетов.

Пользователь удален

Немецкий физик Г.Ом в 1826 году экспериментально установил, что сила тока I, протекающего через однородный металлический проводник (т. Е. Проводник, в котором не действуют внешние силы), пропорциональна напряжению U на концах проводника:

где R = const .
Величину R обычно называют электрическим сопротивлением. Проводник, имеющий электрическое сопротивление, называется резистором. Это соотношение выражает закон Ома для однородного участка цепи: ток в проводнике прямо пропорционален приложенному напряжению и обратно пропорционален сопротивлению проводника.
В СИ единицей измерения электрического сопротивления проводников является ом (Ом). Сопротивлением 1 Ом обладает такой участок цепи, в котором при напряжении 1 В возникает ток 1 А.
Проводники по закону Ома называются линейными. Графическая зависимость тока I от напряжения U (такие графики называются вольт-амперными характеристиками, сокращенно ВАХ) изображается в виде прямой, проходящей через начало координат. Следует отметить, что существует множество материалов и устройств, не подчиняющихся закону Ома, например, полупроводниковый диод или газоразрядная лампа.Даже для металлических проводников при достаточно больших токах наблюдается отклонение от линейного закона Ома, так как электрическое сопротивление металлических проводников увеличивается с ростом температуры.
Для участка цепи, содержащего ЭДС, закон Ома записывается в следующей форме:
IR = U12 = φ1 — φ2 + E = Δφ12 + E.
Это соотношение обычно называют обобщенным законом Ома.
На этом рис. изображает замкнутую цепь постоянного тока. Участок цепи (cd) однородный.

закон Ома,
IR = Δφcd.
Участок (ab) содержит источник тока с ЭДС, равной E.
Согласно закону Ома для неоднородной области
Ir = Δφab + E.
Суммируя оба равенства, получаем:
I (R + r) = Δφcd + Δφab + E.
Но Δφcd = Δφba = — Δφab.
Вот почему

Эта формула выражает закон Ома для замкнутой цепи: ток в замкнутой цепи равен электродвижущей силе источника, деленной на сумму сопротивлений однородной и неоднородной частей цепи.2) | uo io — амплитуды тока и напряжения, r- активное сопротивление цепи, в скобках и в квадрате — реактивная составляющая, sqrt = квадратный корень ….

Оля Семенова

Закон Ома — это эмпирический физический закон, который определяет взаимосвязь между электродвижущей силой источника (или электрическим напряжением) с силой тока, протекающего в проводнике, и сопротивлением проводника. Установлен Георгом Омом в 1826 году и назван в его честь.

В 1826 году немецкий ученый Георг Ом экспериментально установил прямую пропорциональную зависимость между током I в проводнике и напряжением U на его концах: электрическая проводимость проводника … Обратная величина проводимости называется электрическим сопротивлением проводник R … Таким образом, закон Ома для участка цепи, не содержащего источника ЭДС , имеет вид. Учитывая, что в общем случае участок цепи может содержать и ЭДС, закон Ома следует представить в виде.

Сопротивление проводника зависит от его размера, формы и материала, из которого он сделан. Для однородного линейного проводника, где l — длина, S — площадь поперечного сечения проводника, r — удельное электрическое сопротивление, в зависимости от материала, из которого изготовлен проводник. Единица сопротивления 1 Ом — это сопротивление такого проводника, в котором протекает ток 1А при напряжении 1В.

Если цепь замкнута, то, где R — полное сопротивление всей цепи, включая сопротивление источника ЭДС.Тогда следует записать закон Ома для замкнутой цепи , где e — алгебраическая сумма всех ЭДС, имеющихся в этой цепи.

Сопротивление источника тока r — принято называть внутренним , а сопротивление остальной цепи R — внешним … Окончательный вид формулы закона Ома для замкнутой цепи. В единицах СИ напряжение и э.д.с. измеряются в Вольтах (В), сопротивление — в Ом (Ом), удельное электрическое сопротивление — в Ом-метрах (Ом × м), удельная электропроводность — в Сименсах (S).

Закон Ома можно также записать для плотности тока. Рассмотрим сечение электрической длины d l и сечения dS (рисунок 2.1). Сила тока в этой области, сопротивление, падение напряжения, где E — напряженность электрического поля в проводнике. Подставляя эти параметры в закон Ома для участка цепи, получаем … Отсюда или, где — электропроводность проводника или удельная проводимость … В векторной форме имеем (единица измерения g в системе СИ — Сименс на метр (См / м)). Результирующее выражение представляет собой закон Ома в дифференциальной форме: плотность тока в любой точке внутри проводника прямо пропорциональна напряженности поля в этой точке.

1,14 Сопротивление проводника. Явление сверхпроводимости.

Способность вещества проводить ток характеризуется его удельной проводимостью г или удельным сопротивлением р.Их значение определяется химической природой проводника и условиями, в частности температурой, при которой он находится. Для большинства металлов r растет с температурой примерно линейно:, — удельное сопротивление при 0 ° С, t — температура по шкале Цельсия, a — температурный коэффициент сопротивления, близкий к 1/273 K -1 при не очень низких температурах. Поскольку R ~ r, то где — сопротивление при 0 ° С. Преобразовав две последние формулы, можно записать и, где T — температура Кельвина.На основе температурной зависимости сопротивления металлов термометров сопротивления — термисторов , позволяющих определять температуру с точностью до 0,003 К.

При низких температурах линейность зависимости сопротивления металлов от температуры нарушается, а при температуре 0 К наблюдается остаточное сопротивление R покой. Величина R rest зависит от чистоты материала и наличия в нем механических напряжений. Только в идеально чистом металле с идеально правильной кристаллической решеткой R находится в точке T®0 (пунктирная часть кривой).

Кроме того, в 1911 г. Г. Каммерлинг-Оннес обнаружил, что при T k = 4,1 К сопротивление ртути резко падает почти до нуля. Эта температура была названа критической , а наблюдаемое явление — сверхпроводимость … Впоследствии этот эффект был обнаружен в ряде других металлов (Ti, A l , Pb, Zn, V и т. Д.) И их сплавы в интервале температур 0,14-20 К. Вещества в сверхпроводящем состоянии обладают необычными свойствами.Возбужденный в них ток может долгое время существовать без источника тока. Переход в сверхпроводящее состояние сопровождается резким изменением теплоемкости, теплопроводности и магнитных свойств вещества. Оказалось, что внешнее магнитное поле не проникает в толщу сверхпроводника, т.е. магнитная индукция внутри него всегда равна нулю. Явление сверхпроводимости объясняется на основе квантовой теории. На сегодняшний день это явление обнаружено и в ряде композиционных веществ (например, соединения металлов и диэлектриков), критическая температура которых достигает температуры ожижения азота, что позволяет экономично использовать явление высокотемпературного сверхпроводимость в инженерной практике.Это явление позволяет создавать: системы передачи без потерь электрического тока по проводам из таких веществ, системы аккумулирования электричества, мощные электромагниты, магнитные подвески различного назначения.

1.15 Работа и мощность тока, закон Джоуля-Ленца.

Определим работу, совершаемую постоянным током в проводнике с сопротивлением R и под напряжением … Поскольку ток — это движение заряда q под действием поля, работу тока можно определить по формуле.С учетом формулы и закона Ома получаем, или, или, где t — время протекания тока. Разделив обе части равенства на t, получим выражения для power DC N

Работа по току в единицах СИ измеряется в дулях (Дж), а мощность — в ваттах (Вт). На практике также используются внесистемные единицы текущей работы: ватт-час (Вт × ч) и киловатт-час (кВт × ч). 1Вт × ч — работа током мощностью 1Вт в течение одного часа. 1Вт × в = 3,6 × 10 3 Дж.

Опыт показывает, что ток всегда вызывает некоторый нагрев проводника.Нагрев происходит из-за того, что кинетическая энергия электронов, движущихся по проводнику (т. Е. Энергия тока) при каждом столкновении с ионами металлической решетки, превращается в тепло Q. Если ток течет по неподвижному металлическому проводнику, тогда вся работа тока тратится на его нагрев и, следуя закону сохранения энергии, можно записать … Эти соотношения выражают закон Джоуля-Ленца … Этот закон впервые был установлен эмпирически Д. Джоуля в 1843 г. и независимо Э.Ленцем в 1844 году. Использование теплового воздействия тока в технике началось с открытия в 1873 году русским инженером А. Ладыгиным лампы накаливания .

На тепловом воздействии тока основан ряд электрических устройств и установок: термоэлектрические измерительные приборы, электропечи, электросварочное оборудование, бытовые электронагревательные приборы — чайники, котлы, утюги. В пищевой промышленности широко применяется метод электрического контактного нагрева, заключающийся в том, что электрический ток, проходящий через продукт с определенным сопротивлением, вызывает его равномерный нагрев.Например, для производства сосисок через дозатор фарш поступает в формы, торцевые стенки которых служат электродами. Такая обработка обеспечивает равномерный нагрев по всему объему продукта, возможность поддержания определенного температурного режима, максимальную биологическую ценность продукта, минимальную продолжительность процесса и энергозатраты.

Определим удельную тепловую мощность тока Вт, т.е. количество тепла, выделяемого в единице объема за единицу времени.Выделим в проводнике элементарный цилиндрический объем dV сечением dS и длиной d l , параллельным направлению тока и сопротивления ,. Согласно закону Джоуля-Ленца, тепло будет выделяться в этом объеме за время dt. Затем, используя закон Ома для плотности тока и отношения, получаем … Эти отношения выражают закон Джоуля-Ленца в дифференциальной форме .

1,16. Правило Кирхгофа для разветвленных электрических цепей.

До сих пор мы рассматривали простейшие электрические схемы, состоящие из одной замкнутой неразветвленной цепи. Во всех его разделах текущие силы одинаковы. Расчет I, R, e в такой цепочке производится по законам Ома.

Более сложным является разветвленная электрическая схема , состоящая из нескольких замкнутых контуров с общими участками. В каждой цепи может быть несколько источников тока. Токи в отдельных участках замкнутого контура могут быть разными по величине и направлению (рисунок 2.2). В 1847 г. Г. Кирхгоф сформулировал два правила, значительно упрощающих расчет разветвленных цепей.

Первое правило Кирхгофа : алгебраическая сумма токов в узле равна нулю: . Узел — точка цепи, в которой сходятся не менее трех проводников. В электрической схеме на рисунке 2.2 есть два узла A и B. Ток, входящий в узел, считается положительным, а исходящий ток — отрицательным.Например, для узла A следует написать первое правило Кирхгофа.

Первое правило выражает закон сохранения электрического заряда, поскольку они не могут появиться или исчезнуть в любой точке цепи.

Второе правило Кирхгофа относится к любому замкнутому контуру, выделенному в разветвленной цепи: алгебраическая сумма произведений токов и сопротивлений, включая внутренние, во всех секциях замкнутого контура равна алгебраической сумме электродвижущих сил происходит в этом цикле … Путь — это замкнутая часть диаграммы, по которой вы можете пройти и вернуться в исходную точку. Второе правило Кирхгофа получается из закона Ома, написанного для всех участков от узла до узла (ов) разветвленной цепи. На рисунке 2.2 электрическая цепь состоит из трех цепей: AMNBA, CABDC, CMNDC. В этом случае токи I i в ветвях контура, совпадающие с произвольно выбранным направлением обхода контура, считаются положительными, а направленные в сторону обхода — отрицательными.E.m.f. переданные от «+» к «-» считаются положительными и наоборот. В рассматриваемой электрической схеме (рис. 2.2) выбираем обход цепей по часовой стрелке и записываем для них уравнения по правилу II Кирхгофа: для AMNBА; для CABDC; для CMNDC … В этом примере не учитываются внутренние сопротивления источников тока. Первое и второе правила Кирхгофа позволяют вам составить систему линейных алгебраических уравнений, которые связывают параметры (I, R,) и позволяют, зная одни, находить другие.

Простые электрические схемы имеют большое практическое значение. В повседневной жизни полезно знать, как подключить колонки или проигрыватель винила к стереосистеме, как подключить охранную сигнализацию или магнитолу автомобиля, как зарядить аккумуляторы или зажечь елку.

Большинство электрических цепей содержат комбинацию последовательно или параллельно соединенных резисторов (резистор — это элемент цепи, который имеет только сопротивление). Общее сопротивление участка цепи определяется отношением падения напряжения на нем к величине тока.При последовательном соединении (рис. 2.3 а) через все резисторы протекает одинаковый ток. При параллельном подключении (рисунок 2.3 b) полный ток равен сумме токов, протекающих в отдельных резисторах.

При последовательном подключении падение напряжения на участке AB составляет, то есть сумма падений напряжения на трех резисторах. Разделим обе части равенства на I и получим, т.е. … Таким образом, полное сопротивление участка цепи, состоящей из последовательно соединенных резисторов, равно их алгебраической сумме.

При параллельном включении (рис. 2.3 б) имеем … Делим обе части равенства на U, где U — падение напряжения на участке цепи АБ, и, и получаем … Это равенство означает … Обратная величина импеданса параллельно соединенных резисторов равна алгебраической сумме значений их обратных сопротивлений.

Регулируемое (изменяющееся с помощью специального ползунка) сопротивление, которое называется реостатом … По своему назначению реостаты делятся на пусковые, которые служат для ограничения силы тока при пуске двигателя, и регулирующие — для регулировки силы тока в цепи (постепенное уменьшение освещенности в залах театра), регулировки скорости вращения электродвигателей и т. д.Реостат может использоваться как так называемый датчик перемещения … В автоматических регуляторах уровня жидкости в резервуарах используется поплавковый датчик реостата. К двигателю реостата прикреплен специальный поплавок. Изменение уровня жидкости смещает поплавок, изменяет сопротивление реостата, а, следовательно, и ток в цепи, значение которого дает информацию об уровне.

Рассмотрим простейшую замкнутую цепь, состоящую из источника (гальванического элемента, батареи или генератора)

и резистор с сопротивлением (рис.161). Источник тока имеет и сопротивление Сопротивление источника часто называют внутренним сопротивлением, а не сопротивлением внешней цепи. В генераторе это сопротивление обмоток, а в гальваническом элементе сопротивление раствора электролита и электродов

Закон Ома для замкнутой цепи связывает ток в цепи, ЭДС и импеданс цепи. Эта связь может быть установлена ​​теоретически, если использовать закон сохранения энергии и закон Джоуля-Ленца (9.17).

Пусть заряд проходит через поперечное сечение проводника во времени. Тогда работу внешних сил по перемещению заряда можно записать следующим образом: Согласно определению силы тока Следовательно

При выполнении этой работы на внутренние и внешние участки цепи, сопротивление которых представляет собой определенное количество выделяемого тепла. По закону Джоуля-Ленца оно равно:

Согласно закону сохранения энергии, Приравнивая (9.20) и (9.21) получаем:

Произведение силы тока и сопротивления участка цепи часто называется падением напряжения на этом участке. Таким образом, ЭДС равна сумме падений напряжения на внутреннем и внешнем участках замкнутой цепи.

Обычно закон Ома для замкнутой цепи записывается в виде:

Ток в замкнутой цепи равен отношению ЭДС цепи к ее сопротивлению.

Сила тока зависит от трех величин: сопротивления и внешнего и внутреннего участков цепи. Внутреннее сопротивление источника тока не оказывает заметного влияния на силу тока, если оно мало по сравнению с сопротивлением внешней части цепи. В этом случае напряжение на выводах источника примерно равно

Но при коротком замыкании ток в цепи определяется именно внутренним сопротивлением источника и может быть очень большим при электродвижущей силе в несколько вольт, если ее недостаточно (например, с омной батареей).Провода могут расплавиться, а сам источник — выйти из строя.

Если в цепи несколько последовательно соединенных элементов с, то полная ЭДС схемы равна алгебраической сумме ЭДС отдельных элементов. Чтобы определить знак ЭДС любого источника, необходимо предварительно договориться о выборе положительного направления обхода цепи. На рисунке 162 направление обхода против часовой стрелки считается положительным (произвольно).

Если при обходе цепи они переходят от отрицательного полюса источника к положительному, то внешние силы внутри источника в этом случае совершают положительную работу.Если при обходе цепи они переходят от положительного полюса источника к отрицательному полюсу, ЭДС будет отрицательной. Внешние силы внутри источника совершают негативную работу. Итак, для схемы, показанной на рисунке 162:

Если, то согласно (9.23) сила тока, т.е. направление тока совпадает с направлением обхода контура. Напротив, направление тока противоположно направлению обхода контура. Общее сопротивление цепи равно сумме всех сопротивлений:

При параллельном соединении гальванических ячеек с одинаковой ЭДС (или другими источниками) ЭДС аккумулятора равна ЭДС одной из ячеек (рис. .163). Внутреннее сопротивление батареи рассчитывается по обычному правилу параллельного соединения проводов. Для схемы, показанной на рисунке 163, согласно закону Ома для замкнутой цепи, сила тока определяется по следующей формуле:

1. Почему электрическое поле заряженных частиц (кулоновское поле) не может поддерживать постоянный электрический ток в цепи? 2. Что называют внешними силами? 3. Что называется электродвижущей силой?

4.Сформулируйте закон Ома для замкнутой цепи. 5. От чего зависит знак ЭДС в законе Ома для замкнутой цепи?

Невозможно организовать циркуляцию заряда в замкнутом контуре под действием только электростатической силы. Для переноса заряда в область высокого потенциала (2- b -1) вы должны использовать неэлектростатических сил . Такие силы называются внешними силами. Любые силы, кроме электростатических, могут действовать как внешние силы.Устройства, в которых внешние силы действуют на электрические заряды, называются источниками тока. В батареях, например, внешние силы возникают в результате химической реакции взаимодействия электродов с электролитом, в генераторах внешние силы — это силы, действующие на заряды, движущиеся в магнитном поле, и т. Д. Это происходит в источниках тока из-за В результате работы внешних сил создается генерируемая энергия, которая затем расходуется в электрической цепи.

Работа, совершаемая внешними силами при перемещении одиночного положительного заряда, является одной из основных характеристик источника, его электродвижущей силы e:

Поле внешних сил, а также электростатическое поле характеризуется вектором напряженности:

Электродвижущая сила источника равна работе, совершаемой внешними силами при перемещении одиночного положительного заряда по замкнутому контуру.

На участке цепи 1-а-2 движение носителей заряда происходит под действием только электростатической силы = q. Такие участки называют однородными.

Участок замкнутого контура, на который помимо электростатической силы действуют внешние силы, называется неоднородным.

Можно показать, что на однородном участке цепочки средняя скорость направленного движения носителей заряда пропорциональна действующей на них силе.Для этого достаточно сравнить полученные в последней лекции формулы: = (6.3) и = l

Пропорциональность скорости силе и плотности тока интенсивности сохранится в случае неоднородного участка цепи. Но теперь напряженность поля равна сумме напряжённостей электростатического поля и поля внешних сил:.

Это уравнение закона Ома в локальной дифференциальной форме для неоднородного участка цепи.

Теперь перейдем к закону Ома для неоднородного участка цепи в интегральной форме.

Для замкнутого контура уравнение закона Ома несколько видоизменяется, так как разность потенциалов в этом случае равна нулю :.

В законе Ома для замкнутой цепи (7.8) R — полное сопротивление цепи, которое складывается из внешнего сопротивления цепи R 0 и внутреннего сопротивления источника r: R = R 0 + r .

12) Закон Джоуля-Ленца в дифференциальной и интегральной формах.

Пусть в участке электрической цепи I … Напряжение U на концах этого участка численно протекает постоянный ток, равный работе, совершаемой электрическими силами при движении одиночного положительного заряда по этому участку. Это следует из определения напряжения.

Отсюда работа A = q × U … В течение т заряд будет передан по площадке q = I × т и при этом работа будет выполнена : A = q × U = U × I × t .

Это выражение работы электрического тока справедливо для любого проводника.

Произведенная за единицу времени работа — это мощность электрического тока :.

Работа электрического тока (6.14) может быть потрачена на нагрев проводника, выполнение механической работы (электродвигатель) и на химическое действие тока при его протекании через электролит (электролиз).

Если химическое воздействие и механическая работа при протекании тока не производятся, то вся работа электрического тока тратится только на нагрев проводника: Q = A = U × I × t = I 2 × R × т .(6,15)

Закон о тепловом действии электрического тока (6.15) был экспериментально установлен независимо английским ученым Д. Джоулем и российским академиком Э. Ленц. Формула (6.15) — математическая запись закона Джоуля-Ленца в интегральной форме , позволяющая рассчитать количество тепла, выделяемого в проводнике.

.

Перед нами Закон Джоуля-Ленца в дифференциальной форме .

Учитывая, что i = l E =, это выражение также можно записать так:

Правила Кирхгофа.

Рассмотренные нами законы постоянного тока позволяют рассчитывать токи в сложных разветвленных электрических цепях. Эти расчеты упрощены с помощью правил Кирхгофа.

Существует два правила Кирхгофа : текущее правило и правило напряжения .

Правило токов относится к узлам цепи, то есть к таким точкам цепи, где сходятся как минимум три проводника (рис. 7.4.). Правило токов гласит: алгебраическая сумма токов в узле равна нулю:

При составлении соответствующего уравнения текущие в узел токи принимают со знаком плюс, а выходящие из него — со знаком минус.Это первое правило Кирхгофа является следствием уравнения неразрывности (см. (6.7)) или закона сохранения электрического заряда.

Стресс-правило относится к любому замкнутому контуру разветвленной цепи.

Правило напряжений формулируется следующим образом: в любой замкнутой цепи алгебраическая сумма падений напряжения равна алгебраической сумме ЭДС, возникающей в этой цепи:

При составлении уравнения вторые правила Кирхгофа устанавливаются по направлению обхода.

Токи, совпадающие с направлением байпаса, принимают со знаком плюс, токи в обратном направлении — со знаком минус ЭДС. источник принимается со знаком плюс, если он создает ток, совпадающий с направлением байпаса. В противном случае ЭДС отрицательная.

Закон

Ома для замкнутой цепи показывает, что значение тока в реальной цепи зависит не только от сопротивления нагрузки, но и от сопротивления источника.

Формулировка закона Ома для замкнутой цепи звучит так: величина тока в замкнутой цепи, состоящей из источника тока с внутренним и внешним сопротивлением нагрузки, равна отношению электродвижущей силы источника к сумме внутренних и внешние сопротивления.

Впервые зависимость тока от сопротивлений была экспериментально установлена ​​и описана Георгом Омом в 1826 году.

Формула закона Ома для замкнутой цепи записывается следующим образом:

  • I [A] — ток в цепи,
  • ε [V] — ЭДС источника напряжения,
  • R [Ом] — сопротивление всех внешних элементов цепи,
  • r [Ohm] — внутреннее сопротивление источника напряжения

Физический смысл закона

Потребители электрического тока вместе с источником тока образуют замкнутую электрическую цепь.Ток, проходящий через потребителя, также проходит через источник тока, что означает, что ток, помимо сопротивления проводника, является сопротивлением самого источника. Таким образом, полное сопротивление замкнутой цепи будет суммой сопротивления потребителя и сопротивления источника.

Физический смысл зависимости тока от ЭДС источника и сопротивления цепи состоит в том, что чем больше ЭДС, тем больше энергия носителей заряда, а значит, и скорость их упорядоченного движения.С увеличением сопротивления цепи энергия и скорость движения носителей заряда, следовательно, величина тока уменьшаются.

Зависимость можно показать на собственном опыте. Рассмотрим схему, состоящую из источника, реостата и амперметра. После включения в цепи протекает ток, наблюдаемый амперметром, перемещая ползунок реостата, мы увидим, что при изменении внешнего сопротивления изменится ток.

Примеры задач по применению закона Ома для замкнутой цепи

Реостат сопротивлением 4 Ом подключается к источнику ЭДС 10 В и внутренним сопротивлением 1 Ом.Найдите ток в цепи и напряжение на клеммах источника.

Когда к батарее был подключен резистор 20 Ом, ток в цепи был 1 А, а когда был подключен резистор 10 Ом, ток стал 1,5 А. Найдите ЭДС и внутреннее сопротивление батареи.

ТОК СМЕЩЕНИЯ И УРАВНЕНИЯ МАКСВЕЛЛА

ТОК СМЕЩЕНИЯ И УРАВНЕНИЯ МАКСВЕЛЛА

Расчет магнитного поля распределения тока может, в принципа, проводиться с использованием закона Ампера, который связывает интеграл по путям магнитное поле вокруг замкнутого пути к току, перехваченному произвольная поверхность, которая охватывает этот путь:

(35.1)

Закон Ампера не зависит от формы выбранной поверхности, пока ток течет по непрерывной непрерывной цепи. Однако рассмотрим случай в котором токовый провод оборван и подключен к конденсатору с параллельными пластинами (см. рисунок 35.1). Во время зарядки по проводу будет течь ток. процесс конденсатора. Этот ток будет генерировать магнитное поле, и если мы далеко от конденсатора, это поле должно быть очень похоже на магнитное поле, создаваемое бесконечно длинным непрерывным проводом.Однако ток, перехваченный произвольной поверхностью, теперь зависит от выбранной поверхности. Например, поверхность, показанная на рисунке 35.1, не пропускает ток. Ясно, что в этом случае закон Ампера не может быть применен для нахождения магнитного поля. поле генерируется текущим.

Рисунок 35.1. Закон Ампера в конденсаторной цепи. Хотя поверхность, показанная на рисунке 35.1, не пересекает любой ток он перехватывает электрический поток. Допустим, конденсатор идеальный конденсатор, с однородным электрическим полем E между пластинами и без электрическое поле вне пластин.В определенный момент t заряд на обкладок конденсатора — Q. Если обкладки имеют площадь поверхности А, то электрическая поле между пластинами равно

(35,2)

Электрическое поле вне конденсатора равно нулю. Электрический поток, [Phi] E , перехваченный поверхностью, показанной на рисунке 35.1, равен на номер

(35,3)

Если по проводу течет ток I, то заряд конденсатора тарелки будут зависеть от времени.Следовательно, электрический поток также будет временем зависимая, а скорость изменения электрического потока равна

(35,4)

Магнитное поле вокруг провода теперь можно найти, изменив закон Ампера

(35,5)

где [Phi] E — электрический поток через поверхность, обозначенную на Рис. 35.1 В наиболее общем случае поверхность, покрываемая интегрированием путь магнитного поля может перехватывать ток и электрический поток.В таком В этом случае влияние электрического потока и электрического тока должно быть вместе, и закон Ампера становится

(35,6)

Ток I — это ток, улавливаемый любой поверхностью, используемой в расчет, и не обязательно совпадает с током в проводах. Уравнение (35.6) часто записывается как

(35,7)

где I d называется , ток смещения и определяется как

(35.8)

Пример: Задача 35.8

Конденсатор с параллельными пластинами имеет круглые пластины площадью А, разделенные расстояние d. Тонкая прямая проволока длиной d проходит по оси конденсатор и соединяет две пластины. Этот провод имеет сопротивление R. внешние выводы пластин подключены к источнику переменной ЭДС с напряжением V = V 0 sin (Омега) t.

а) Какой ток в тонком проводе?

б) Какой ток смещения через конденсатор?

в) Какой ток поступает на внешние выводы конденсатора?

г) Какое магнитное поле между обкладками конденсатора на расстоянии r от ось? Предположим, что r меньше радиуса пластин.

а) Установку можно рассматривать как параллельную цепь резистора с сопротивление R и конденсатор емкостью C (см. рисунок 35.2). Электрический ток в тонкой проволоке можно получить по закону Ома

(35.9)

Рисунок 35.2. Принципиальная проблема 35.8.

б) Напряжение на конденсаторе равно внешней ЭДС. Электрический поэтому поле между пластинами конденсатора равно

(35.10)

Следовательно, электрический поток через конденсатор равен

. (35.11)

Ток смещения I d можно получить, подставив уравнение (35.11) в уравнение (35.8)

(35.12)

Ток на внешних выводах конденсатора — это сумма тока используется для зарядки конденсатора и тока через резистор. Заряд на конденсаторе

(35.13)

Таким образом, зарядный ток равен

. (35,14)

Таким образом, общий ток равен

. (35.15)

г) Силовые линии магнитного поля внутри конденсатора будут образовывать концентрические круги, с центром вокруг резистора (см. рисунок 35.3). Интеграл по путям магнитное поле вокруг круга радиуса r равно

(35.16)

Рисунок 35.3. Петля Ампера, используемая для определения магнитного поля. внутри конденсатора. Поверхность, которая будет использоваться для определения силы тока и электрического Перехватываемый поток — это диск радиуса r, показанный на рис. 35.3. Электрический поток через этот диск равно

(35.17)

Ток смещения, улавливаемый этой поверхностью, равен

. (35.18)

Ток, перехваченный поверхностью, равен току через резистор (ур.(35.9)). Таким образом, закон Ампера требует

(35,19)

Таким образом, сила магнитного поля равна

. (35.20)

Основные уравнения, описывающие поведение электрических и магнитных поля известны как уравнения Максвелла. Их

(35.21)

(35.22)

(35.23)

(35,24)

Уравнения Максвелла дают полное описание взаимодействия между заряды, токи, электрические и магнитные поля. Все свойства поля могут быть получены путем математических манипуляций с этими уравнениями. Если дано распределение зарядов и токов, то эти уравнения однозначно определить соответствующие поля.

Пример: Задача 35.10

Докажите, что уравнения Максвелла математически предполагают сохранение электрический заряд; то есть доказать, что если электрический ток не течет в При заданном объеме электрический заряд в этом объеме остается постоянным.

Уравнение (35.21) показывает, что вложенный заряд Q связан с электрическим током. поток [Phi] E :

(35,25)

Скорость изменения вложенного заряда может быть определена дифференцированием уравнение (35.25) по времени

(35.26)

Замкнутая поверхность, используемая в уравнении (35.24) для определения потока и тока перехваченный может быть заменен сумкой, рот которой сжался до нуля.Путь поэтому интеграл магнитного поля вдоль устья равен нулю, и уравнение (35.24) можно записать как

(35.27)

Используя уравнение (35.26), мы можем переписать уравнение (35.27) как

(35.28)

Другими словами, если ток не течет в замкнутом объеме и не выходит из него (I = 0) тогда электрический заряд в этом объеме останется постоянным. Из этого следует сохранение заряда.

Электрическое поле между пластинами конденсатора с параллельными пластинами равно определяется внешней ЭДС.Если расстояние между пластинами равно d (см. Рисунок 35.4), то электрическое поле между пластинами равно

(35.29)

Это зависящее от времени электрическое поле будет индуцировать магнитное поле с напряженностью что может быть получено с помощью закона Ампера. Рассмотрим круговую амперовскую петлю из радиус r. Интеграл по траекториям магнитного поля вокруг этой петли равен на номер

(35.30)

Электрический поток через поверхность, охватываемую этим путем, равен

. (35.31)

Рисунок 35.4. Колебательный конденсатор с параллельными пластинами. Таким образом, ток смещения равен

. (35.32)

Используя закон Ампера, получаем для магнитного поля

(35,33)

Это зависящее от времени магнитное поле будет индуцировать электрическое поле. Общая электрическое поле внутри конденсатора, следовательно, будет суммой постоянных электрическое поле, создаваемое источником ЭДС и индуцированного электрического поля, генерируется зависящим от времени магнитным полем.Сила наведенного электрическое поле можно рассчитать с помощью закона индукции Фарадея. Рассмотреть возможность замкнутый путь, показанный на рисунке 35.4. Возьмем индуцированное электрическое поле на оси конденсатора равняется нулю. Интеграл по путям индуцированной электрической поле вдоль указанного пути тогда равно

(35,34)

где E ind считается положительным, если направлено вверх. Магнитный поток через поверхность, охватываемую петлей, показанной на рисунке. 35.4 равно

(35,35)

Таким образом

(35,36)

Индуцированное электрическое поле E ind может быть получено из закона Фарадея. индукции (уравнение (35.23)) и равно

(35,37)

Таким образом, полное электрическое поле равно

. (35,38)

Но добавление индуцированного поля означает, что необходимо внести поправку. приложено к магнитному полю, рассчитанному ранее (ур.(35.33)). Это, в свою очередь, измените индуцированный ток, и этот процесс будет продолжаться вечно. Если мы пренебрегаем дополнительные поправки, то уравнение (35.38) показывает, что электрическое поле исчезает на радиусе R, если

(35,39)

или

(35,40)

Если мы создадим полость, заключив конденсатор в проводящий цилиндр из радиуса R, тогда уравнение (35.40) может быть использовано для определения частоты движения ЭДС, которая вызовет стоячую волну.Эта частота называется резонансной. частота и равна

(35,41)

Для резонатора с R = 0,5 м резонансная частота составляет 1,2 ГГц. Электромагнитное излучение с частотой в этом диапазоне называется микроволновым. излучения, а полость называется микроволновой печью.

Электрическое поле, создаваемое неподвижным зарядом, стационарно. Когда заряд ускоряется, он создает дополнительные электрические и магнитные поля, которые перемещаются наружу из положения заряда.Эти радиационные поля называются электромагнитных волн . Они движутся со скоростью света (в вакуума) и уносят энергию и импульс от заряда. Их свойства определяются свойствами ускоренного заряда, и таким образом предоставить средства для передачи информации со скоростью света в течение длительного времени расстояния.

Рассмотрим сначала заряд q в состоянии покоя (при t <0). Между t = 0 и t = [tau], заряд ускоряется с ускорением a.После t = [tau] заряд движется с постоянной скоростью (v = a [тау]). Будем считать, что конечная скорость заряда мала по сравнению со скоростью света (v << c) и что период времени [тау], в течение которого заряд ускоряется короткий. Электрическое поле, создаваемое зарядом в момент времени t> [тау] состоит из трех отдельных областей (см. рисунок 35.5). В области r> ct силовые линии будут линией точечного заряда, покоящегося в начале координат (электромагнитные волны распространяются со скоростью света, и область с r > ct может еще не знать, что заряд отошел от зародыша).В сферическая область радиусом r

Радиальная составляющая определяется законом Гаусса.Рассмотрим сферическую Гауссова поверхность, расположенная между двумя сферами, показана на рисунке 35.5. В заряд, заключенный на этой поверхности, равен q. Электрический поток через это поверхность зависит только от радиальной составляющей поля. Применение закона Гаусса заключаем, что радиальная составляющая электрического поля — это просто обычная Кулоновское поле

(35.42)

Рисунок 35.5. Силовые линии электрического поля, создаваемые ускоренным плата. Связь между радиальной составляющей электрического поля а поперечная составляющая электрического поля может быть определена как внимательно исследуя одну линию поля (см. рисунок 35.6). Линия поля, показанная на Рисунок 35.6 составляет угол [тета] с направлением движущегося заряда. Соотношение между величиной поперечного электрического поля и радиального электрическое поле равно

(35,43)

Поскольку радиальное поле известно, уравнение.(35.42), мы можем использовать уравнение (35.43) для определения поперечная составляющая электрического поля:

(35,44)

Расстояние r, на котором происходит перегиб, связано со временем t, на котором мы посмотрите на поле:

(35,45)

Исключая зависимость от t в уравнении (35.44), получаем следующее выражение для поперечной компоненты электрического поля:

(35.46)

Рисунок 35.6. Расчет поперечного электрического поля. Уравнение (35.46) показывает, что поперечное электрическое поле равно прямо пропорционально ускорению a и обратно пропорционально расстояние r. Кулоновская составляющая поля падает как 1 / r 2 . Это показывает, что поперечная составляющая, также называемая полем излучения, остается значительным на расстояниях, где кулоновское поле практически исчезает.

Уравнение для поперечного электрического поля (ур.(35.46)) действует в вообще, даже если ускорение не постоянное. Если заряд колеблется назад и вперед простым гармоническим движением с частотой [омега], затем ускорение в момент t будет равно

(35,47)

Чтобы определить поле излучения в момент времени t и на расстоянии r, мы имеем чтобы понять, что ускорение a, используемое в уравнении (35.46), должно быть ускорение в момент времени t — r / c, где r / c — время, необходимое для того, чтобы сигнал путешествовать на расстояние r.Поле излучения колеблющегося заряда равно следовательно, равно

(35,48)

Пример: задача 35.25

На радиоантенне (прямой кусок провода) электроны движутся назад и вперед в унисон. Предположим, что скорость электронов v = v 0 cos ([omega] t), где v 0 = 8,0 x 10 -3 м / с и [омега] = 6,0 × 10 6 рад / с.

а) Какое максимальное ускорение электронов?

б) В соответствии с этим максимальным ускорением, какова сила поперечное электрическое поле, создаваемое одним электроном на расстоянии 1.0 км от антенну в направлении, перпендикулярном антенне? Сколько времени задержка (или замедление) между моментом максимального ускорения и момент, когда соответствующее электрическое поле достигает расстояния 1,0 км ?

в) На антенне 2,0 x 10 24 электронов. Что коллективное электрическое поле, создаваемое всеми электронами, действующими вместе? Предполагать антенна достаточно мала, так что все электроны вносят примерно такое же электрическое поле на расстоянии 1.0 км.

а) Ускорение электронов можно получить, дифференцируя их скорость относительно времени:

(35,49)

Таким образом, максимальное ускорение равно

. (35,50)

б) Максимальное поперечное электрическое поле на расстоянии 1,0 км (= 1000 м), в направление, перпендикулярное антенне ([theta] = 90deg.), может быть получено используя уравнение (35.48):

(35.51)

Поскольку скорость распространения поля излучения равна скорости свет, c, максимумы в поле излучения будут происходить за период времени [Delta] t после максимумов ускорения электрона. Длина этого период [Delta] t равен

(35,52)

в) Если предположить, что все электроны находятся в фазе, то максимальное суммарное поперечное электрическое поле на расстоянии 1,0 км равно количеству электронов, умноженных на максимальное поперечное электрическое поле, создаваемое одним электрон.Таким образом

(35,53)

Когда заряд ускоряется из состояния покоя, он создает магнитное поле. Изначально магнитное поле будет равно нулю (заряд в состоянии покоя). Как в результате ускорения возмущение переместится наружу и изменит магнитное поле от его начального значения (B = 0 Тл) до его конечного значения, в значительной степени так же, как мы наблюдали для электрического поля. Магнитное поле может быть полученный из электрического потока по закону Максвелла-Ампера, который гласит, что

(35.54)

Обратите внимание, что ток I не появляется в ур. (35,54). Поскольку мы ищем в области, удаленной от движущегося заряда, ток, перехваченный поверхностью , охватываемый путем, используемым для вычисления интеграла по путям B, равен нулю. Индуцированное магнитное поле будет зависеть от времени и, следовательно, будет вызывать электрическое поле по закону Фарадея. Это индуцированное электрическое поле снова будет зависит от времени и индуцирует другое магнитное поле, и этот процесс продолжается.Комбинированные электрические и магнитные поля излучения, создаваемые Ускоряющий заряд называют электромагнитными волнами. Они есть хозрасчетные; электрическое поле индуцирует магнитное поле, а индуцированное магнитное поле индуцирует электрическое поле. Поскольку электрические и магнитные поля естественно поддерживают друг друга, электромагнитная волна не требует среда для его распространения, и он легко распространяется в вакууме. Максвелл уравнения могут быть использованы, чтобы показать, что произведение u 0 и [epsilon] 0 равно 1 / c 2 .Или

(35,55)

Это уравнение было одним из величайших и ранних триумфов Максвелла. электромагнитная теория света. Это показывает, что электричество и магнетизм два разных аспекта одного и того же явления.


Отправляйте комментарии, вопросы и / или предложения по электронной почте на адрес [email protected] и / или посетите домашнюю страницу Фрэнка Вольфса.

Несколько вопросов и ответов по теории сетей

1) В любой линейной сети такие элементы, как индуктор, резистор и конденсатор, всегда _________

а. Экспонат изменения в связи с изменением температуры
b. Изменения в экспонате в связи с изменением напряжения
c. Изменения в экспонате в связи с изменением времени
d. Остается постоянным независимо от изменения температуры, напряжения и времени

ОТВЕТ: (d) Остается постоянным независимо от изменений температуры, напряжения и времени

2) Какие типы сетей допускают физическое разделение сетевых элементов (резисторов, катушек индуктивности и конденсаторов) для целей анализа?

а. Сети с сосредоточенными объектами
b. Распределенные сети
c. Односторонние сети
д. Двусторонние сети

ОТВЕТ: (a) Сосредоточенные сети

3) Какой закон играет важную роль в петлевом анализе сети?

а. KCL
б. КВЛ
г. Теорема о законе суперпозиции
d. Ничего из вышеперечисленного

ОТВЕТ: (б) KVL

4) Чем отличается петлевой анализ на уровне приложения / функционирования от закона Кирхгофа?

а. Использование токов контура вместо токов ответвления для записи уравнений
b. Способность тока ветви передавать несколько сетей
c. Уменьшение количества неизвестных для сложных сетей
d. Все вышеперечисленное

ОТВЕТ: (d) Все вышеперечисленное

5) Какая теорема помогает заменить ветвь импеданса в сети другой сетью, состоящей из других компонентов схемы, не влияя на отношения V-I во всей сети?

а. Теорема суперпозиции
б. Теорема компенсации
c. Теорема о подстановке
d. Теорема о максимальной передаче мощности

ОТВЕТ: (а) Теорема о подстановке

6) Что делать, если в цепи присутствуют зависимые источники тока и напряжения при применении «теоремы суперпозиции»?

а. Заменить на обрыв цепи
б. Заменил на короткое замыкание
гр. Сохранять в первоначальном виде без замены при обрыве или коротком замыкании
d. Ничего из вышеперечисленного

ОТВЕТ: Сохраните в первоначальном виде без замены на обрыв или короткое замыкание

7) Какой правильный последовательный порядок действий следует предпринять при применении теоремы Тевенина?

A. Расчет эквивалентного напряжения Тевенина
B. Удаление полного сопротивления ветви, через которое должен быть оценен требуемый ток
C.Оценка эквивалентного импеданса между двумя выводами ответвления
D. Оценка тока ответвления путем схематического изображения эквивалентной схемы Тевенина

а. A, C, B, D
б. B, A, C, D
c. Д, А, В, В
д. B, C, D, A

ОТВЕТ: B, A, C, D

9) Какие из приведенных ниже утверждений точно связаны с условиями, применимыми к тому, что путь является неправильным подграфом?

А.Распространенность одной ветви на оконечном узле
B. Распространенность двух ответвлений на остальных узлах

а. A истинно, а B ложно
b. A ложно, а B верно
c. Оба A и B верны
d. Оба A и B ложны

ОТВЕТ: (c) Оба A и B верны

10) Сколько минимальных конечных узлов или конечных узлов задействовано в дереве в соответствии с его свойствами?

а. Только один
б. Два
c. Четыре
г. бесконечное

ОТВЕТ: (b) Два

11) Каким будет значение прямоугольной (полной инцидентности) матрицы, если связанная ветвь ориентирована в сторону узла?

а. 1
б. -1
г. 0
г. Не определено (∞)

ОТВЕТ: (б) -1

12) Согласно теории линейных графов, количество возможных деревьев всегда равно определителю произведения ______

а. Только полная матрица инцидентности
b. Уменьшенная матрица инцидентности и ее транспонирование
c. Матрица Cut-Set
d. Матрица для галстуков

ОТВЕТ: (b) Уменьшенная матрица инцидентности и ее транспонирование

13) Рассмотрим математическое представление дифференциального уравнения порядка n -го , приведенное ниже. Что в нем означает обозначение v (t)?

a 0 (d n i / dt n ) + a 1 (d n -1 i / dt n-1 ) + …… + a n-1 (di / dt) + a n i = v (t)
a. Независимая переменная
б. Входная или принудительная функция
c. Возбуждение
д. Все вышеперечисленное

ОТВЕТ: (d) Все вышеперечисленное

14) Если дифференциальное уравнение называется однородным, каково будет значение вынуждающей функции?

а. 0
б. 1
г.
г. –1

ОТВЕТ: (а) 0

15) Какое обозначение момента времени означает, что неизменное состояние сети вот-вот изменится?

а. т (0) +
б. т (0)
с. т *
д. т ˆ

ОТВЕТ: (b) t (0)

16) Какие элементы ведут себя как разомкнутые цепи, особенно с учетом постоянного тока? количества?

а. Катушки индуктивности
б. Резисторы
в. Конденсаторы
г. Все вышеперечисленное

ОТВЕТ: (c) Конденсаторы

17) Каким будет значение коэффициента мощности для последовательной цепи RLC при явлении резонанса?

а. 0
б. 0,5
г. 1
г. бесконечность

ОТВЕТ: (c) 1

18) Который из следующих get / s отменен из-за условия резонанса в переменном токе. цепей, если индуктивное и емкостное реактивные сопротивления параллельны?

а. Реактивное сопротивление
б. Принятие
c. Сопротивление
г. Все вышеперечисленное

ОТВЕТ: (b) Приостановление

19) Какое из следующих условий выполняется при резонансе?

а. X c > X L
b. X c = X L
c. X c L
d. Ничего из вышеперечисленного

ОТВЕТ: (b) X c = X L

20) Кривая реактивного сопротивления — это, по сути, график индивидуальных реактивных сопротивлений стихов __________

а. Частота
б. Фаза
c. Амплитуда
д. Период времени

ОТВЕТ: (а) Частота

21) Какой тип импеданса в асимметричной сети оценивается на одной паре сетевых терминалов, особенно в цепочке бесконечных сетей?

а. Импеданс изображения
б. Итерационный импеданс
c. Волновое сопротивление
d. Все вышеперечисленное

ОТВЕТ: (b) Итеративное сопротивление

22) Где в «правильно завершенных асимметричных сетях» отображается терминирование на обоих портах сети?

а. Импеданс изображения
b. Итерационные импедансы
c. In характеристическое сопротивление
d. Все вышеперечисленное

ОТВЕТ: (a) Импеданс изображения

23) Какая единица измерения используется для измерения вносимых потерь?

а. Непер
б. Вебер
г. Ом
г. Вт

ОТВЕТ: (а) Непер

24) Как вносимые потери представлены в виде отношения мощностей?

а. α = 10 log 10 | P 1 / P 2 | децибел
б. α = 10 log 10 | P 2 / P 1 | децибел
c. α = 20 log 10 | P 1 / P 2 | децибел
г. α = 20 log 10 | P 2 / P 1 | децибел

ОТВЕТ: (a) α = 10 log 10 | P 1 / P 2 | децибел

25) Какой тип фильтра показан ниже?

а. Фильтр нижних частот
б. Фильтр высоких частот
c. Полосовой фильтр
d. Ленточный элиминационный фильтр

ОТВЕТ: (d) Ленточный устраняющий фильтр

26) При разработке фильтра нижних частот с постоянным k (Т-образное сечение), показанного ниже, каким будет значение конденсатора, если L / 2 = 20 мГн, R 0 = 500 Ом и f c = 5 кГц ?

а. 0,0635 мкФ
б. 0.10 мкФ
c. 0,1273 мкФ
г. 0,20 мкФ

ОТВЕТ: (c) 0,1273 мкФ

27) Какое будет значение затухания в полосе задерживания на частоте 8 кГц для ФНЧ постоянного типа k с Т-образным сечением с частотой среза около 4 кГц?

а. 10,03 дБ
б. 22,87 дБ
c. 35,04 дБ
г. 50,02 дБ

ОТВЕТ: (b) 22,87 дБ

28) Что касается характеристик π-секционного фильтра нижних частот, приведенных ниже, каков будет фазовый сдвиг на 2 кГц в полосе пропускания?

а. 0,7731 радиан
б. 1.0471 радиан
c. 2,551 радиан
г. 3.991 радиан

ОТВЕТ: (b) 1.0471 радиан

29) Какие типы аттенюаторов обеспечивают фиксированную величину ослабления, позволяя пользователю изменять ослабление в несколько шагов?

а. Лестничные аттенюаторы
б. Аттенюаторы с переменным значением
c. Падовые аттенюаторы
d. Все вышеперечисленное

ОТВЕТ: (a) Лестничные аттенюаторы

30) Для показанного ниже симметричного Т-образного аттенюатора, каково значение сопротивления шунтирующего плеча при N = 10 и проектном импедансе = 500 Ом?

а. 51,21 Ом
б. 101,01 Ом
c. 305,90 Ом
г. 409,90 Ом

ОТВЕТ: (b) 101,01 Ом

31) Примите во внимание приведенные ниже утверждения.Что из них не является недостатком преобразования Лапласа?

а. Непригодность для обработки данных при случайных колебаниях
б. Анализ дискретных входов
c. Возможность преобразования s = jω только для анализа синусоидального установившегося состояния
d. Неспособность существовать для нескольких функций распределения вероятностей

ОТВЕТ: (b) Анализ дискретных входов

32) Что означает «σ» в уравнении комплексной частотной переменной s = σ + jω при определении преобразования Лапласа?

А.Константа затухания
B. Коэффициент затухания
C. Константа распространения
D. Фазовая постоянная

а. A&B
б. C&D
г. B&C
г. B&D

ОТВЕТ: (a) A&B

33) Каким образом «σ» комплексной частотной переменной появляется во временной области?
а. В качестве линейной мощности
б. В качестве реактивной мощности
c. В экспоненциальной степени
d. Как итеративная мощность

ОТВЕТ: (c) В экспоненциальной степени

34) На графической схеме, показанной ниже, какое уравнение поведения во временной области будет получено из-за комплексной частотной переменной для σ> 0?

а. e σt sin ωt
b. e σt cos ωt
c. e σt sin ωt + e σt cos ωt
d. e σt sin ωt — e σt cos ωt

ОТВЕТ: (b) e σt cos ωt

35) В цепи полное сопротивление управляющей точки составляет (с + 5) / (с 2 + 5 с + 1).Следовательно, входная точка схемы составляет __________

.

а. (с + 5)
б. (s 2 + 5s + 1)
c. (s + 5) x (s 2 + 5s + 1)
d. 2 + 5 с + 1) / (с + 5)

ОТВЕТ: (d) (s 2 + 5s + 1) / (s + 5)

36) Вычислите передаточную функцию отношения напряжений для сети, показанной ниже. Предположим, что ток, протекающий через ответвление между узлами «a» и «Q», равен (I a + 4 I a ) i.е. 5I a

а. -3/7
б. -5/11
г. -3/11
г. -5/7

ОТВЕТ: (b) -5/11

37) Передаточная проводимость — это отношение преобразований Лапласа ________

а. Ток на одном порту относительно напряжения на другом порту
b. Напряжение на одном порту относительно тока на другом порту
c. Ток в одном порту относительно тока в другом порту
d. Напряжение в одной точке относительно напряжения в другом порту

ОТВЕТ: (a) Ток на одном порту относительно напряжения на другом порту

38) Когда сетевая функция выражается как отношение выходных преобразований Лапласа к входным переменным системы, тогда она рассматривается как _______

а. Системная функция
б. Передаточная функция
c. И a, и b
d. Ничего из вышеперечисленного

ОТВЕТ: (c) И a, и b

39) Как называется подключение источника энергии к порту сети?

а. Driving Point
б. Пункт передачи
c. И a, и b
d. Ничего из вышеперечисленного

ОТВЕТ: (a) Driving Point

40) Для сети, представленной ниже, каким будет значение ‘z 22 ‘, если оценочные z-параметры равны ‘z 11 ‘ = 30 и ‘z 12 ‘ = ‘z 21 ‘= 10?

а. 10
б. 20
г. 30
г. 40

ОТВЕТ: (d) 40

41) Каково потенциальное значение опорного или опорного узла, используемого при узловом анализе сети?

а. Ноль
б. Юнити
г. Больше нуля, но меньше бесконечности
d. непредсказуемо

ОТВЕТ: (а) Ноль

42) Какой будет стоимость эквивалентного одиночного источника, если два источника напряжения, соединенные последовательно, имеют одинаковую или одинаковую полярность?

а. Добавление двух источников с полярностями, аналогичными полярности двух источников
b. Добавление двух источников с противоположной полярностью
c. Разница между двумя источниками с полярностью, аналогичной полярности двух источников
d. Разница между двумя источниками с полярностью, аналогичной полярности большей среди двух источников

ОТВЕТ: (a) Добавление двух источников с полярностями, аналогичными полярности двух источников

43) Каким должно быть значение сопротивления на схеме, показанной ниже?

а. Ноль
б. Infinity
c. Юнити
г. Невозможно предсказать

ОТВЕТ: (а) Ноль

44) Каким должно быть значение I 2 в соответствии с методом тока ячейки для приведенной ниже принципиальной схемы?

а. 1,39 А
б. 1,63 А
г. 2,33 А
г. 5 А

ОТВЕТ: (b) 1.63 A

45) Что из следующего также считается «двойственным к теореме Тевенина»?

а. Теорема Нортона
б. Теорема суперпозиции
c. Теорема Миллмана
d. Теорема о максимальной передаче мощности

ОТВЕТ: (а) Теорема Нортона

46) Какова была бы природа «Z L », если бы реактивное сопротивление «Z eq » было индуктивным согласно «теореме о максимальной передаче мощности»?

а. Индуктивный
б. Емкостный
c. резистивный
d. Все вышеперечисленное

ОТВЕТ: (б) Емкостный

47) Какое значение импеданса нагрузки (Z L ) необходимо подключить к клеммам R-S для максимальной передачи мощности в сети, показанной ниже?

а. 5 + j5 Ом
б. 5 — j5 Ом
c. 10 + j10 Ом
d. 10 — j10 Ом

ОТВЕТ: (d) 10 — j10 Ω

48) Каково значение импеданса «Z 2 » согласно теореме Миллера для сети, показанной ниже?

а. З.К / К-1
б. Z / 1-K
г. З / К-1
г. З-1 / К

ОТВЕТ: (a) Z.K / K-1

49) Каким будет количество деревьев, если граф демонстрирует сокращение в виде приведенной ниже сокращенной матрицы инцидентов?

а. 16
г. 24
г. 26
г. 28

ОТВЕТ: (b) 24

50) Сколько фундаментальных сечений будет сгенерировано для графа с n числом узлов?

а. н + 1
б. н-1
с. n 2 (n-1)
д. н / н-1

ОТВЕТ: (b) n-1

51) Какой параметр должен по существу равняться количеству узлов в сети в соответствии с принципом двойственности?

а. Полный импеданс
б. Общий доступ
c. Количество ячеек
д. Количество источников напряжения

ОТВЕТ: (c) Количество ячеек

52) Рассмотрите приведенные ниже утверждения.Какой из них имеет / не указывает / обладает свойством «Полная матрица заболеваемости»?

а. Определитель цикла полной матрицы инцидентности всегда равен нулю
b. Сумма всех записей в любом столбце никогда не должна равняться нулю
c. Ранг связного или ориентированного графа всегда равен «n-1»
d. Все вышеперечисленное

ОТВЕТ: (b) Добавление всех записей в любой столбец никогда не должно равняться нулю

53) Какой тип сетевого ответа проявляет свое поведение с учетом связанного с ним эффекта времени?

а. Нулевой входной отклик
b. Ответ нулевого состояния
c. И a, и b
d. Ничего из вышеперечисленного

ОТВЕТ: (b) Ответ нулевого состояния

54) Какой вывод можно сделать из приведенной ниже диаграммы, показывающей характеристики напряжения на резисторе и катушке индуктивности в зависимости от времени, в связи с экспоненциальным ростом тока через катушку индуктивности?

а. Напряжение на резисторе и катушке индуктивности увеличивается
b. Напряжение на резисторе и катушке индуктивности уменьшается
c. Напряжение на резисторе увеличивается, но уменьшается на катушке индуктивности
d. Напряжение на резисторе уменьшается, но увеличивается на катушке индуктивности

ОТВЕТ: (c) Напряжение на резисторе увеличивается, но уменьшается на катушке индуктивности

55) Рассмотрим схему, изображенную ниже. Каким будет значение i (0 ), особенно если индуктор действует как короткое замыкание?

а. 0,3 А
б. 2 А
г. 5 А
г. 10 А

ОТВЕТ: (b) 2 A

56) Сколько секунд потребуется, чтобы текущий i (t) стал половиной своего начального значения после t = 0 в нарисованной ниже сети

а. 866 мкс
б. 1039 мкс
c. 1200 мкс
д. 1849 мкс

ОТВЕТ: (b) 1039 мкс

57) Как ведет себя последовательный резонансный контур в условиях резонанса?

а. Усилитель тока
б. Transconductance
c. Регулятор напряжения
г. Усилитель напряжения

ОТВЕТ: (d) Усилитель напряжения

58) Если переменный ток генератор сигналов управляет последовательной цепью RLC, тогда цепь подвергается резонансу только из-за изменения __________

а. Напряжение питания
б. Сопротивление серии
c. Частота питания
d. Фазовый угол

ОТВЕТ: (c) Частота сети

59) Напряжение питания токоподводов, если в последовательном резонансном контуре работает __________ резонансная частота

а. Выше
б. Ниже
ок. Равно
d. Ничего из вышеперечисленного

ОТВЕТ: (b) Ниже

60) Каким будет значение импеданса параллельного резонансного контура в условиях антирезонанса?

а. Резистивный и максимальный
б. Резистивный и минимальный
c. Реактивное и максимальное
d. Реактивный и минимум

ОТВЕТ: (a) Резистивное и максимальное

61) Каким будет характеристическое сопротивление тройника для симметричной сети, показанной ниже?

а. 300,15 Ом
б. 529,15 Ом
гр. 715,15 Ом
г. 900,15 Ом

ОТВЕТ: (b) 529,15 Ом

62) Рассмотрим симметричную Т-образную сеть, показанную ниже. Каким будет значение постоянной распространения, если характеристическое сопротивление оценивается равным Z 0 = 850,64 Ом?

а. 0,2824 непер
б. 0,1412 непер
с. 0,0706 непер
д. 0,0353 непер

ОТВЕТ: (б) 0,1412 непер

63) Рассмотрим нижеприведенные предположения о двух симметричных Т-сетях в каскадной конфигурации.Какое будет значение тока, протекающего через оконечное сопротивление для конфигурации сети, показанной ниже?

Допущения:
n = -2,
? = 1,5,
I с = 0,6 A

а. 0,0112 А
б. 0,0298 А
в. 0,04145 А
г. 0,0812 А

ОТВЕТ: (b) 0,0298 A

64) Если сеть состоит исключительно из резистивных элементов, что она даст из следующего?

а. Затухание
б. Фазовый сдвиг
c. И a, и b
d. Ничего из вышеперечисленного

ОТВЕТ: (а) Затухание

65) Из чего обычно состоят фильтры верхних частот?

A. Емкостное плечо
B. Емкостное плечо
C. Индуктивное плечо
D. Индуктивное плечо

а. A&D
б. A&C
с. B&C
г. B&D

ОТВЕТ: (a) A&D

66) Каким будет значение индуктора при проектировании прототипа Т-образной секции фильтра верхних частот, если расчетное сопротивление и частота среза составляют 600 Ом и 1500 Гц соответственно?

а. 19,89 мГн
б. 31,83 мГн
c. 40,13 мГн
г. 51,83 мГн

ОТВЕТ: (b) 31,83 mH

67) В полосовом элиминирующем фильтре частота резонанса отдельных плеч геометрическая _________

а. Среднее значение двух частот среза
b. Разница двух частот среза
c. Произведение двух частот среза
d. Разделение двух частот среза

ОТВЕТ: (a) Среднее значение двух частот среза

68) Какое значение ‘m’ выбрано в составном фильтре при подключении оконечных секций для получения надлежащего согласования импеданса и постоянного характеристического импеданса во всей полосе пропускания?

а. 0,3
б. 0,6
г. 0,9
г. 0,12

ОТВЕТ: (б) 0,6

69) Ссылаясь на схему симметричного π-аттенюатора ниже, какое будет расчетное значение затухания в непере, если D = 20 дБ & R 0 = 400 Ом?

а. 10
б. 20
г. 40
г. 80

ОТВЕТ: (а) 10

70) Почему регулируемые аттенюаторы применимы для радиовещания?

а. Для регулирования скорости
b. Для регулировки громкости
c. Для контроля времени
д. Для регулировки мощности

ОТВЕТ: (b) Для регулировки громкости

71) Со ссылкой на диаграмму, показанную ниже, на сколько моментов функция линейного изменения смещается или задерживается?

а. Т
б. т — Т
г. 1
г.

ОТВЕТ: (а) Т

72) Рассмотрим функцию f (t), которая удовлетворяет приведенному ниже дифференциальному уравнению.Какое уравнение получится, если взять преобразование Лапласа и заменить члены f (0 ) и f ‘(0 ) на ноль?

[d 2 f (t) / dt 2 ] + 5 [df (t) / dt] + 6 f (t) = 10

а. [S 2 F (s) + 5s F (s) +6 F (s)] = 10 / s
b. [S 2 F (s) + 5s F (s) — 6 F (s)] = 10 / s
c. [S 2 F (s) — 5s F (s) + 6 F (s)] = 10 / s
d. [S 2 F (s) — 5s F (s) — 6 F (s)] = 10 / s

ОТВЕТ: (a) [S 2 F (s) + 5s F (s) +6 F (s)] = 10 / s

73) В соответствии с теорией области Лапласа трансформная проводимость сопротивления составляет _______

а. Импеданс
б. Поведение
c. Индуктивность
d. Емкость

ОТВЕТ: (b) Поведение

74) Каким будет значение импеданса преобразования одиночного конденсатора в области Лапласа на схеме, показанной ниже?

а. 1 / sC
б. СК
г. СК — 1
г. SC / 1+ SC

ОТВЕТ: (a) 1 / sC

75) Что представляет собой значение константы «k» в приведенной ниже факторизованной форме сетевого уравнения?
H (s) = [k (s — z 1 ) (s — z 2 ) —– (s — z m )] / [(s — p 1 ) (s — p ) 2 )….. (s — p n )]

A. Коэффициент усиления системы
B. Коэффициент масштабирования
C. Коэффициент вектора
D. Коэффициент качества системы

а. A&B
б. C&D
г. A&C
г. B&D

ОТВЕТ: (a) A&B

76) Если функция сложной системы является аналитической по своей природе, точки в s-плоскости рассматриваются как ________

а. Обычные точки
б. Особые точки
c. Несколько точек
d. Все вышеперечисленное

ОТВЕТ: (а) Обычные баллы

77) Какие из следующих относятся к категории критической частоты?

а. поляков
б. Нули
с. И a, и b
d. Ничего из вышеперечисленного

ОТВЕТ: (c) И a, и b

78) Каково идеальное значение сетевой функции на полюсах?

а. Ноль
б. Юнити
г. Infinity
г. Конечное и ненулевое

ОТВЕТ: (c) Infinity

79) Какие элементы действуют как независимые переменные в Y-параметрах?

а. Текущий
б. Напряжение
c. И a, и b
d. Ничего из вышеперечисленного

ОТВЕТ: (b) Напряжение

80) Что из нижеперечисленного считается допуском прямого перехода при коротком замыкании?

а. л 11
б. л 12
ок. л 21
д. л 22

ОТВЕТ: (c) y 21

81) При каких условиях метод преобразования источника остается напрямую неприменимым?
а. Только при отсутствии полного сопротивления последовательно с источником напряжения
b. Только при отсутствии полного сопротивления параллельно источнику тока
c. И a, и b
d. Ничего из вышеперечисленного

ОТВЕТ: (c) И a, и b

82) Если в сети присутствуют в параллельной конфигурации бесчисленное количество ветвей, какой метод считается наиболее полезным для анализа сети?

а. Узловой метод
б. Метод сетки
c. И a, и b
d. Ничего из вышеперечисленного

ОТВЕТ: (a) Метод узла

83) Какая операция может быть выполнена или выполнена в соответствии с теоремой Миллмана с точки зрения преобразования источников напряжения или тока в один эквивалентный источник напряжения или тока?

а. Вычитание
б. Комбинация
c. Дифференциация
г. Интеграция

ОТВЕТ: (b) Комбинация

84) Почему теорема суперпозиции неприменима к власти?

а. Потому что он пропорционален квадрату тока, а ток является нелинейной функцией
b. Потому что оно пропорционально квадрату напряжения, а напряжение является нелинейной функцией
c. И a, и b
d. Ничего из вышеперечисленного

ОТВЕТ: (a) Потому что он пропорционален квадрату тока, а ток является нелинейной функцией

85) Каким будет порядок матрицы импеданса ветвей для приведенного ниже уравнения равновесия KVL на основе анализа петли или сетки?

E = B (V с — Z b I с )

а. б x 1
б. b x b
c. б-н + 1) х 1
д. (б-н + 1) х б

ОТВЕТ: (b) b x b

86) Согласно теории графов петлевого анализа, сколько уравнений равновесия требуется на минимальном уровне с точки зрения количества ветвей (b) и количества узлов (n) в графе?

а. н-1
б. б + (п-1)
в. б- (н-1)
д. б / н-1

ОТВЕТ: (c) b- (n-1)

87) Какова природа корней для незатухающих цепей с устойчивыми колебаниями?

а. Чисто мнимое
б. Действительное, равное и отрицательное
c. Комплексное сопряжение с отрицательной действительной частью
d. Действительное, неравное и отрицательное

ОТВЕТ: (а) Чисто воображаемый

88) Каким будет время, необходимое отклику для достижения 50% своего значения в установившемся состоянии для системы второго порядка?

а. π — θ / ω d
б. π / ω d
в. 4 / ξ ω n
д. (1+ 0,7 ξ) / ω n

ОТВЕТ: (d) (1+ 0,7 ξ) / ω n

89) Какова будет природа импеданса на частоте ниже антирезонансной частоты?

а. Емкостный
б. Индуктивный
c. Реактивный
д. резистивный

ОТВЕТ: (б) Индуктивная

90) Если значение резонансной частоты составляет 50 кГц в последовательной цепи RLC вместе с полосой пропускания около 1 кГц, то каково значение коэффициента качества?

а. 5
б. 50
г. 100
г. 500

ОТВЕТ: (b) 50

91) Каким будет значение постоянной затухания, особенно для сети, состоящей из чисто реактивных элементов?

а. 0
б. 1
г. -1
г. бесконечность

ОТВЕТ: (а) 0

92) Предположим, что сеть состоит из чисто резистивных элементов, какое будет значение постоянной распространения (генерируемый выходной сигнал) в терминах постоянной затухания и фазовой постоянной из следующего?

а. γ = α + j0
б. γ = 0 + jβ
в. γ = 0 — jβ
д. γ = α — j0

ОТВЕТ: γ = α + j0

93) Можно преодолеть недостаток фильтра, полученного из m, путем соединения ряда секций в дополнение к секциям прототипа и m-типа с оконечной нагрузкой __________

а. Четвертые участки
б. Половинки
c. Площадь трех четвертых участков
д. Полные разделы

ОТВЕТ: (b) Половинки

94) Какое идеальное значение затухания для частот в полосе пропускания, особенно для каскадной конфигурации?

а. Ноль
б. Юнити
г. Infinity
г. непредсказуемо

ОТВЕТ: (а) Ноль

95) Переменные аттенюаторы демонстрируют переменное затухание, но постоянное __________

а. Входное сопротивление
б. Выходное сопротивление
c. И a, и b
d. Ничего из вышеперечисленного

ОТВЕТ: (c) И a, и b

96) В соответствии со схемой мостового аттенюатора с переменным ‘T’ ниже, сколько резисторов следует изменить, чтобы добиться переменного затухания?

а. Только один
б. Два
c. Три
г. Четыре

ОТВЕТ: (b) Два

97) Если значение (P 1 / P 2 ) в соотношении мощностей, выраженное в дБ, больше единицы, что означает «D» в сети?

а. Потеря мощности
б. Прирост мощности
c. Стабильность мощности
d. Энергосбережение

ОТВЕТ: (a) Потеря мощности

98) Каким будет импеданс управляющей точки сети домена Лапласа, показанной ниже?

а. (10 с 2 + 5,25 с + 1,5) / (с + 0,125)
б. (6s 2 + 2,25s + 1,5) / (s + 0,25)
c. (5s 2 + 2,25s + 1,5) / (s + 0,16)
д. (8 с 2 + 2,25 с + 1,5) / (с + 0,8)

ОТВЕТ: (a) (10s 2 + 5,25s + 1,5) / (s + 0,125)

99) Рассмотрим схему RC, показанную ниже. Каким будет значение постоянной времени для RC-цепи?

а. 0,25 с
б. 0,50 с
c. 1 сек
д. 2 секунды

ОТВЕТ: (c) 1 сек

100) Каким будет значение i (t), особенно при t> 0 в схеме RC, показанной ниже? [Предположим, что полярности v c (0 ) аналогичны падению на конденсаторе из-за i (t)]

а. -2,125 e -1т A
b. -4,125 e -2t A
c.-8 e -4t A
d. -10 e -5 лет A

ОТВЕТ: (a) -2,125 e -1t A

101) Для графика полюс-ноль, приведенного ниже, каким будет значение системной функции H (s), если постоянный ток усиление системы 20?

а. 10 (с + 4) / (с + 1) (с 2 + 6 с — 10)
б. 20 (с — 4) / (с + 3) (с 2 _ 6 с +10)
с. 40 (с + 4) / (с + 3) (с 2 — 6 с — 10)
д. 50 (с + 4) / (с + 1) (с 2 + 6 с +10)

ОТВЕТ: (d) 50 (s + 4) / (s + 1) (s 2 + 6s +10)

102) Какие колебания будут генерироваться в ответе во временной области, если присутствуют комплексно сопряженные полюса с отрицательной действительной частью?

а. Затухающие колебания
б. Незатухающие колебания
c. Устойчивые колебания
d. Ничего из вышеперечисленного

ОТВЕТ: (a) Затухающие колебания

103) Какие из нижеперечисленных случаев ответственны за генерацию колебаний с возрастающей амплитудой в ответе системной функции во временной области?

А.Сложные полюсы с положительной действительной частью
B. Сложные полюсы с отрицательной действительной частью
C. Повторяющиеся полюса на мнимой оси
D. Повторяющиеся полюсы на действительной оси

а. A&C
б. B&D
г. B&C
г. A&D

ОТВЕТ: (a) A&C

104) Как рассчитывается полная проводимость при обратном переносе при коротком замыкании (y 12 ) в терминах отношения тока и напряжения?

а. V 2 / I 1 (сохраняя I 2 = 0)
b. I 2 / V 1 (сохраняя V 2 = 0)
c. I 1 / V 2 (сохраняя V 1 = 0)
d. В 1 / I 2 (сохраняя I 1 = 0)

ОТВЕТ: (c) I 1 / V 2 (сохраняя V 1 = 0)

105) Коэффициент усиления обратного напряжения холостого хода в h-параметрах является безразмерной величиной и обычно эквивалентен ________

а. V 1 / I 1 (сохраняя V 2 = 0)
b. I 2 / I 1 (сохраняя V 2 = 0)
c. V 1 / V 2 (сохраняя I 1 = 0)
d. I 2 / V 2 (сохраняя I 1 = 0)

ОТВЕТ: (c) V 1 / V 2 (сохраняя I 1 = 0)

106) Какое правильное условие симметрии наблюдается в z-параметрах?

а. z 11 = z 22
б. z 11 = z 12
c. z 12 = z 22
г. z 12 = z 21

ОТВЕТ: (a) z 11 = z 22

107) Что из следующего представляет точное условие взаимности для параметров передачи?

а. AB — CD = 1
б. г. н.э. — до н.э. = 1
г. AC — BD = 1
д. Ничего из вышеперечисленного

ОТВЕТ: (b) AD — BC = 1

108) Если два порта соединены в каскадной конфигурации, то какая арифметическая операция должна выполняться между отдельными параметрами передачи, чтобы определить общие параметры передачи?

а. Дополнение
б. Вычитание
c. Умножение
д. Отдел

ОТВЕТ: (c) Умножение

109) Если z-параметры равны z 11 = 40, z 22 = 50 и z 12 = z 21 = 20, каким будет значение y 22 в матричной форме y-параметры, указанные ниже?

а. 4/160
б. 5/160
г. 10/160
г. 15/150

ОТВЕТ: (а) 4/160

Multisim черный ящик

Приварка на поворотной оси

Дизайн подвесного потолка для спальни 2019

No Kode Buku Judul Buku Penerbit Pengarang Tahun Terbit Sumber; 1: 0001/00 / BO / 2015-001: BUKU LENGKAP RAGAM PSIKOTES KERJA: DIVA: AKHMAD FATHONI: 2011: BOPTN: 2: 0001/00 / BO / 2015-002 Инженеры по аппаратному обеспечению тестируют, разрабатывают исследования и проектируют компьютерные системы и компоненты такие как устройства памяти, сети, процессоры и маршрутизаторы.Инженеры по аппаратному обеспечению отображают следующие роли и обязанности в успешных резюме — проектирование нового компьютерного оборудования, чертежей компьютерного оборудования, тестирование завершенной модели компьютерного оборудования, обновление существующих компьютерных систем, чтобы оно … Изучите 112 762 фотографии RTV Oost на Flickr!

Цифровые операции наклона

Несмотря на то, что определение направления вращения является основной задачей при установке двигателя, оно имеет решающее значение для работы связанной нагрузки.

Chevy s10 no power

— Файлы справки: справка на испанском языке для серии 74 (Kike_Gl).Исправления ошибок: — Степпер: ограничивающая рамка перекрывает контакты. — Шаговый двигатель не обновляется после изменения количества шагов. — Проблема с Arduino 1.8.10 решена. Shift Register — бесплатно скачать в формате Powerpoint Presentation (.ppt), PDF-файл (.pdf), текстовый файл (.txt) или просмотреть слайды презентации в Интернете. adaa

Самая длинная серия рейдов Osrs

Инженеры по аппаратному обеспечению тестируют, разрабатывают исследования и проектируют компьютерные системы и компоненты, такие как устройства памяти, сети, процессоры и маршрутизаторы. Инженеры по аппаратному обеспечению отображают следующие роли и обязанности в успешных резюме — проектирование нового компьютерного оборудования, чертежей компьютерного оборудования, тестирование завершенной модели компьютерного оборудования, обновление существующих компьютерных систем, чтобы оно соответствовало…

Значения повреждений пакета ресурсов Minecraft

Multisim — это приложение для захвата и моделирования схем из пакета National Instruments Circuit Design Suite, набора инструментов EDA (Electronics Design Automation), который помогает вам выполнять основные этапы процесса проектирования схем. . Multisim разработан для схемного ввода, моделирования и передачи на следующие этапы, такие как разводка печатной платы. При поддержке Texas Instruments: Процесс моделирования может повысить уровень уверенности в ваших проектах.Как только проблемы будут выявлены и исправлены с помощью SPICE, затем создайте …

Mazda 3, шум детонации

MultiSIM BLUE позволяет создавать схемы, моделировать, компоновку печатных плат, спецификации и приобретать все в одном интегрированном инструменте. Сравните все остальное. Ни один другой БЕСПЛАТНЫЙ инструмент не даст вам такого уровня интеграции. Разъемы, межкомпонентные соединения — прямоугольные соединители — разъемы, розетки, розетки есть в наличии у DigiKey. Заказать сейчас! Connectors, Interconnects отгружают в тот же день. Опытный специалист по инспектированию с подтвержденным опытом работы в сфере экологических услуг.Имеет опыт работы с Microsoft Word, NI Multisim, Xilinx, Matlab и Microsoft Excel. Сильный специалист по операциям с техником-электронщиком, специализирующимся в области электроники из Centennial College. 1 год опыта сбора.

Обновление микропрограммы Hughes ht2000w

Shift Register — бесплатная загрузка в формате Powerpoint Presentation (.ppt), PDF-файл (.pdf), текстовый файл (.txt) или просмотр слайдов презентации в Интернете. adaa

Nissan qg13 руководство по двигателю

• Использованы методы тестирования «черный ящик» / «белый ящик» для поиска ошибок и проблем в коде.Эти методы также использовались для оптимизации окончательного кода… • Разработал систему управления отелем с использованием Java, которая помогла пользователю забронировать номер в отеле, выбрать время и выполнить многие другие функции, которые удовлетворяли бы все общие потребности клиента. Страница: 2 4.5) Разработайте комбинационную схему с тремя входами, x, y и z, и тремя выходами, A, B и C. Когда двоичный вход равен 0, 1, 2 или 3, двоичный выход равен единице. больше, чем вход.

Системный блок Echo 0 mmcblk0boot0 force_ro

NI Multisim (ранее MultiSIM) — это программа для захвата и моделирования электронных схем, которая является частью набора программ проектирования схем, наряду с NI Ultiboard.Multisim — одна из немногих программ для проектирования схем, в которой используется оригинальное программное моделирование на основе Berkeley SPICE. Концепции организационных изменений будут связаны с организационными принципами, которым следует Woolworths Group в первом задании. Изменения — важнейшие аспекты повышения эффективности управления.

монет Wattpad

• Директор центра SMart, доктор Элиза Шипон-Блюм (доктор Э), первой предложила эффективные методы лечения избирательного мутизма.• Программа доктора Э. «Лечение тревожности в социальных сетях» (S-CAT®) признана золотым стандартом для лечения СМ и все чаще используется при лечении ряда других проблем социального общения. Просмотрите профиль Шалахи Панчбая в LinkedIn, крупнейшем в мире профессиональном сообществе. В профиле Shalakha указано 4 вакансии. Просмотрите полный профиль в LinkedIn и узнайте о связях Шалахи и его работе в аналогичных компаниях.

Lexus is300 проблемы с автоматической коробкой передач

Misalnya jika hanya diperlukan data tentang response tegangan-arus dalam rentang batas pendek, maka LED dapat diperlakukan sebagai sebuah black box.Берикутня получает информацию об Интернете отдельно от Google и Bing, используется для изменения компонента светодиодной информации. Ваш мультиметр должен иметь два проводных щупа с красной и черной маркировкой. Они изолированы пластиком, имеют металлические наконечники и обычно имеют форму ручки. Черный зонд подключается к вашему мультиметру …

Задние фонари Genesis coupe канада

NI Multisim (ранее MultiSIM) — это программа для захвата и моделирования электронных схем, которая является частью набора программ для проектирования схем вместе с NI Ultiboard.Multisim — одна из немногих программ для проектирования схем, в которой используется оригинальное программное моделирование на основе Berkeley SPICE.

Формула закона Ома для замкнутой электрической цепи. Закон сохранения заряда

В этой статье мы поговорим о законе Ома, формулах для полной цепи (замкнутой), участка цепи, неоднородного участка цепи, в дифференциальной и интегральной форме, переменного тока, а также для магнитопровода. . Вы узнаете, какие материалы соответствуют и не соответствуют закону Ома, а также где это происходит.
: постоянный ток, протекающий по проводнику, прямо пропорционален напряжению, приложенному к его концам, и обратно пропорционален сопротивлению.

Закон Ома был сформулирован немецким физиком и математиком Георгом Омом в 1825-26 годах на основе опыта. Это экспериментальный закон, а не универсальный — он применим к определенным материалам и условиям.

Закон Ома является частным случаем более позднего и более общего — второго закона Кирхгофа

Ниже будет видео, объясняющее закон Ома на пальцах.

Формула закона Ома для участка цепи

Сила постоянного тока, протекающего по проводнику, пропорциональна напряжению, приложенному к его концам. В Интернете эту формулу часто называют первым законом Ома:

U- напряжение

I- сила (интенсивность) тока

R — Сопротивление

Электрическое сопротивление:

Коэффициент пропорциональности R называется электрическим сопротивлением или сопротивлением.

Отношение напряжения к току для данного проводника постоянно:

Единица электрического сопротивления 1 Ом (1 Ом):

Сопротивление резистора равно 1, если приложенное напряжение составляет 1 вольт, а ток — 1 ампер.

Зависимость электрического сопротивления от размера направляющей:

Сопротивление токопроводящего участка с постоянной площадью поперечного сечения R прямо пропорционально длине этого участка li, обратно пропорционально площади поперечного сечения S:

R- электрическое сопротивление

ρ — удельное сопротивление

I- длина направляющей

S- площадь поперечного сечения

Это соотношение было экспериментально подтверждено британским физиком Хамфри Ди в 1822 году до разработки закона Ома.

Закон Ома для замкнутой (полной) цепи

— это величина силы (силы) тока в этой цепи, которая зависит от сопротивления нагрузки и источника тока (E), его также называют вторым законом Ома.

Лампочка является потребителем источника тока, соединяя их вместе, они образуют законченную электрическую цепь. На картинке выше вы можете увидеть полную электрическую схему, состоящую из батареи и лампы накаливания.

Электричество проходит через лампу накаливания и через саму батарею. Следовательно, ток, проходящий через лампу, будет дальше проходить через батарею, то есть сопротивление лампочки добавляется к сопротивлению батареи.

Сопротивление нагрузки (лампочка), называется внешнее сопротивление , а сопротивление источника тока (батареи) внутреннее сопротивление … Сопротивление батареи обозначается латинской буквой r.

Когда электричество течет по цепи, внутреннее сопротивление самого элемента сопротивляется протеканию тока, и поэтому тепловая энергия теряется в самом элементе.

  • E = электродвижущая сила в вольтах, В
  • I = ток в амперах, А
  • R = сопротивление нагрузки в цепи в Ом, Ом
  • r = внутреннее сопротивление ячейки в Ом, Ом

Мы можем изменить это уравнение;

Это уравнение появляется ( В, ), то есть конечная разность потенциалов , измеренная в вольтах (В).Это разность потенциалов на выводах ячейки при протекании тока в цепи, она всегда меньше ЭДС. клетки.

Закон Ома для неоднородного участка цепи

Если на участок цепи действуют только потенциальные силы (, рис. 1а, ), то закон Ома записывается в известной форме. Если же действие внешних сил также проявляется в круге ( Рис. 2b ), то закон Ома принимает вид, где. Это закон Ома для любой части цепи..

Закон Ома можно распространить на весь круг. Соединив точки 2 и 1 ( Рисунок 3c ), преобразуем разность потенциалов в ноль, и с учетом сопротивления источника тока закон Ома примет вид. Это выражение закона Ома для полной цепи .

Последнее выражение может быть представлено в различных формах. Как известно, напряжение во внешней секции зависит от нагрузки, то есть
или, или.

В этих выражениях Ir — это падение напряжения внутри источника тока, и также видно, что напряжение U меньше ε на величину Ir … Причем, чем больше внешнее сопротивление по сравнению с внутренний, тем больше U приближается к ε.

Рассмотрим два особых случая, касающихся внешнего сопротивления цепи.

1) R = 0 — это явление называется коротким замыканием.Тогда из закона Ома имеем -, то есть ток в цепи увеличивается до максимума, а падение внешнего напряжения U 0. В этом случае в источнике выделяется большая мощность, что может привести к его неисправности.

2) R = ∞ , то есть электрическая цепь разорвана, значит, но … Итак, в данном случае ЭДС численно равна напряжению на выводах открытого источника тока.

Закон Ома в дифференциальной форме

Закон Ома можно представить таким образом, чтобы он не зависел от размера проводника.Выбрать сечение проводника Δ л , , на концах которого приложены потенциалы φ 1 и φ 2. При средней площади поперечного сечения проводника Δ S и плотности тока Дж , то сила тока

Если Δ l → 0, затем взяв предел отношения, … Итак, окончательно получаем, либо в векторной форме — это выражение закона Ома в дифференциальной форме … Этот закон выражает силу тока в произвольной точке проводник в зависимости от его свойств и электрического состояния.

Закон Ома для переменного тока

Это уравнение записано по закону Ома для цепей переменного тока относительно их значений амплитуды. Понятно, что это будет справедливо и для действующих значений силы и тока:.

Для цепей переменного тока возможен случай, когда, то есть U L = U C … Поскольку эти напряжения в противофазе, они компенсируют друг друга.Такие условия называются , резонанс напряжения … Резонанс может быть достигнут либо при ω = const при изменении СО и L , либо при постоянных СО и L выберите ω, который называется резонансным . … Как видно — .

Характеристики резонанса напряжения следующие:

Наконец, из (2) — (4) имеем интегральное выражение для закона Ома

, который он установил экспериментально.

Толкование закона Ома

Сила тока, которая является действием приложенного напряжения, изменяется пропорционально его напряжению. Например: если приложенное напряжение удваивается, это также удваивает силу тока (сила тока).

Помните, что закону Ома удовлетворяет только подмножество материалов — в основном металлы и керамика.

Когда соблюдается закон Ома и какие материалы соответствуют и не соответствуют закону Ома?

Закон Ома — это экспериментальный закон, который выполняется для некоторых материалов (например, металлов) при фиксированных условиях тока, в частности температуры проводника.

Материалы закона

Ома называются омическими проводниками или линейными проводниками. Примерами проводников, соответствующих закону Ома, являются металлы (например, медь, золото, железо), определенная керамика и электролиты.

Неомные материалы, сопротивление которых зависит от силы тока, протекающего через них, называются нелинейными проводниками. Примерами руководящих принципов, не относящихся к закону Ома, являются полупроводники и газы.

Закон Ома не выполняется при изменении параметров проводника, особенно температуры.

Соединенный проводами с различными электроприборами и потребителями электрической энергии, он образует электрическую цепь.

Электрическую цепь принято изображать с помощью схем, на которых элементы электрической цепи (сопротивления, источники тока, переключатели, лампы, устройства и т. Д.) Отмечены специальными значками.

Направление тока в цепи — это направление от положительного полюса источника тока к отрицательному.Это правило было установлено в 19 веке. и с тех пор наблюдается. Движение реальных зарядов может не совпадать с условным направлением тока. Итак, в металлах отрицательно заряженные электроны являются носителями тока, и они движутся от отрицательного полюса к положительному, то есть в противоположном направлении. В электролитах реальное движение зарядов может совпадать или быть противоположным направлению тока, в зависимости от того, какие ионы являются носителями заряда — положительные или отрицательные.

Включение элементов в электрическую цепь может быть последовательным или параллельным .

Закон Ома для полной цепи.

Рассмотрим электрическую схему, состоящую из источника тока и резистора R .

Закон

Ома для замкнутой цепи устанавливает взаимосвязь между током в цепи, ЭДС и общим сопротивлением цепи, состоящим из внешнего сопротивления R и внутреннего сопротивления источника тока R .

Работа внешних сил A st источник тока, согласно определению ЭДС ( ɛ ) равно A st = ɛq , где q — заряд, вытесненный ЭДС. По определению действующего q = It , где t — время, в течение которого был передан заряд. Отсюда имеем:

А ул = ɛ Это .

Тепло, выделяемое при выполнении работы в контуре, согласно закону Джоуля-Ленца , равно:

Q = I 2 Rt + I 2 РТ .

По закону сохранения энергии A = Q … Приравнивая ( A st = ɛ Это ) и ( Q = I 2 Rt + I 2 rt ) получаем:

ɛ = ИК + Ir.

Закон Ома для замкнутой цепи обычно записывается как:

.

Ток в полной цепи равен отношению ЭДС цепи к ее полному сопротивлению.

Если в цепи несколько источников, соединенных последовательно с ЭДС 1 , ɛ 2 , ɛ 3 и т. Д., То полная ЭДС схемы равна алгебраической сумме ЭДС отдельных источников. Знак ЭДС источника определяется относительно направления обхода контура, которое выбрано произвольно, например, на рисунке ниже — против часовой стрелки.

Внешние силы внутри источника делают положительную работу. И наоборот, для цепочки справедливо следующее уравнение:

ɛ = ɛ 1 + ɛ 2 + ɛ 3 = | ɛ 1 | — | ɛ 2 | — | ɛ 3 | …

В соответствии с силой тока положительная с положительной ЭДС — направление тока во внешней цепи совпадает с направлением обхода контура. Суммарное сопротивление цепи с несколькими источниками равно сумме внешнего и внутреннего сопротивлений всех источников ЭДС, например, для цифры выше:

R n = R + r 1 + r 2 + r 3.

Содержимое:

Каждый специалист, занимающийся ремонтом и обслуживанием электроустановок, должен хорошо знать и применять на практике закон Ома для замкнутой цепи. Это правда, поскольку закономерности, открытые немецким физиком Георгом Омом, лежат в основе всей электротехники. Этот закон стал весомым вкладом в дальнейшее развитие научных знаний в области электричества.

Физические свойства закона Ома

Прямая зависимость между силой тока и напряжением, подаваемым в сеть, была обнаружена Омом в 1826 году.Позже понятие напряжения было заменено более точным термином — электродвижущей силой (ЭДС). После теоретического обоснования этой зависимости был выведен закон для замкнутой цепи. Его важная особенность — обязательное отсутствие какого-либо внешнего возмущения. Поэтому стандартные составы потеряют актуальность, если, например, поместить проводник в переменное магнитное поле.

Для опытов по выводу закона использовалась простейшая схема, состоящая из источника питания с ЭДС и двух подключенных к нему клемм, подключенных к резистору.Элементарные частицы, несущие заряд, начинают двигаться в проводнике в определенном направлении. Таким образом, оно представляется как отношение ЭДС к общему сопротивлению всей цепи: I = E / R.

В представленной формуле E — это электродвижущая сила, измеренная в вольтах, I — ток в амперах, а R действует как электрическое сопротивление резистора, измеренное в омах. При этом учитываются все составляющие сопротивления и в расчетах используется их суммарное значение.К ним относятся сопротивление самого резистора, проводника (r) и источника питания (r0). Окончательная формула будет выглядеть так: I = E / (R + r + r0). Если значение внутреннего сопротивления источника тока r0 превышает сумму R + r, то в этом случае отсутствует зависимость силы тока от характеристик подключаемой нагрузки, и источник ЭДС играет роль источника тока. . Когда r0 меньше суммы R + r, получается обратная пропорция тока к общему внешнему сопротивлению, и напряжение поступает от источника питания.

Закон Ома для проведения расчетов

Для точных расчетов необходимо учитывать все потери напряжения, в том числе в точках подключения. Для определения электродвижущей силы на выводах источника тока измеряется разность потенциалов при разомкнутой цепи, когда нагрузка полностью отключена. В этом случае применяется не только закон Ома для замкнутой цепи, но и действующий закон. Этот участок считается однородным, так как здесь учитывается только разность потенциалов без учета ЭДС.Это дает возможность рассчитать каждый элемент электрической цепи по формуле I = U / R, в которой U — разность потенциалов или напряжение, измеренное в вольтах.

Измерения выполняются с помощью вольтметра при подключении щупов к клеммам нагрузки или сопротивления. Результирующее значение напряжения всегда будет ниже электродвижущей силы. Это наиболее распространенная формула, позволяющая найти любой компонент при наличии двух известных.

Закон Ома для замкнутой цепи имеет много общего с законом, полученным для магнитной цепи. В этой системе проводник выполнен в виде замкнутого магнитопровода. Источником является обмотка катушки, по виткам которой течет электрический ток. Возникающий магнитный поток (F) замыкается на магнитную цепь и начинает циркулировать по цепи. Он напрямую зависит от магнитодвижущей силы и сопротивления материала, через который он проходит. Это явление выражается формулой Ф = F / Rm, в которой F — магнитодвижущая сила, а Rm — сопротивление затухания.

Как рассчитать цепи

Закон Ома для замкнутой цепи говорит об этом. Величина тока в замкнутой цепи, которая состоит из источника тока с внутренним сопротивлением, а также внешнего сопротивления нагрузки. Будет равно отношению электродвижущей силы источника к сумме внешнего и внутреннего сопротивлений.

Формула 1 — Закон Ома для замкнутой цепи

Где R Сопротивление внешней цепи измеряется в Ом

r внутреннее сопротивление источника тока также измеряется в омах

I Сила тока в цепи.Измеряется в амперах

E Электродвижущая сила источника тока измеряется в вольтах

Иногда возникают ситуации, когда необходимо найти ток в цепи, но напряжение на ее концах не выставлено. Тем не менее сопротивление цепи и электродвижущая сила источника тока известны. В этом случае применить закон Ома для участка цепи невозможно.

В данном случае применяется закон Ома для замкнутой цепи. Чтобы выяснить принцип действия этого закона, проведем эксперимент.Для этого нам понадобится источник тока, реостат, вольтметр и амперметр.

Для начала построим схему, состоящую из источника тока реостата и амперметра. Перед началом эксперимента установите реостат в максимальное положение. После включения в цепи появится ток, который можно будет наблюдать амперметром. Перемещая ползунок реостата, мы увидим, что при изменении внешнего сопротивления цепи изменяется ток.

Рисунок 1 — Измерение тока в цепи

Затем, оставив на реостате определенное сопротивление, подключите другое такое же сопротивление параллельно источнику тока.И мы увидим, что ток в цепи увеличится. Казалось бы, у обоих источников одинаковое напряжение, сопротивление внешней цепи не изменилось, почему ток увеличился.

Это произошло из-за уменьшения внутреннего сопротивления источника тока. А так как в замкнутой цепи он включен последовательно с внешним сопротивлением и источником тока. Тогда это внутреннее сопротивление также участвует в образовании тока в цепи.

Формула 2 — это закон Ома для замкнутой цепи с числом параллельно подключенных n источников тока.

На основании вышеизложенного можно сделать вывод, что в реальной замкнутой электрической цепи значение тока не способно бесконечно увеличиваться при возникновении короткого замыкания в источнике тока, так как это значение ограничено внутренним сопротивлением источника тока.

Замкнутый контур (рис. 2) состоит из двух частей — внутренней и внешней. Внутренняя часть цепи — источник тока с внутренним сопротивлением r ; внешний — различные потребители, соединительные провода, устройства и т. Д.Суммарное сопротивление внешней части обозначается как R … Тогда полное сопротивление цепи составляет R + R .

Закон Ома для внешнего участка цепи 1 → 2 имеем:

\ (~ \ varphi_1 — \ varphi_2 = IR. \)

Внутренний участок цепочки 2 → 1 неоднороден. Согласно закону Ома \ (~ \ varphi_2 — \ varphi_1 + \ varepsilon = Ir \). Складывая эти равенства, получаем

\ (~ \ varepsilon = IR + Ir.\ qquad (1) \)

\ (~ I = \ frac (\ varepsilon) (R + r). \ Qquad (2) \)

Последняя формула — это закон Ома для замкнутой цепи постоянного тока. Ток в цепи прямо пропорционален ЭДС источника и обратно пропорционален общему сопротивлению цепи .

Поскольку для однородного участка цепи разность потенциалов равна напряжению, то \ (~ \ varphi_1 — \ varphi_2 = IR = U \) и формулу (1) можно записать:

\ (~ \ varepsilon = U + Ir \ Rightarrow U = \ varepsilon — Ir.\)

Из этой формулы видно, что напряжение на внешнем участке уменьшается с увеличением тока в цепи при ε = конст.

Подставляя силу тока (2) в последнюю формулу, получаем

\ (~ U = \ varepsilon \ left (1 — \ frac (r) (R + r) \ right). \)

Давайте проанализируем это выражение для некоторых предельных режимов работы схемы.

а) С разомкнутой цепью ( R → ∞) U = ε , г.е. напряжение на полюсах источника тока при разомкнутой цепи равно ЭДС источника тока.

Это основа возможности примерного измерения ЭДС источника тока с помощью вольтметра, сопротивление которого намного больше внутреннего сопротивления источника тока (\ (~ R_v \ gg r \)) . Для этого к клеммам источника тока подключают вольтметр.

б) Если к выводам источника тока подсоединен проводник, сопротивление которого \ (~ R \ ll r \), то R + r r , то \ (~ U = \ varepsilon \ left (1 — \ frac (r) (r) \ right) = 0 \), а текущее \ (~ I = \ frac (\ varepsilon) (r) \) — достигает максимального значения.

Подключение проводника с незначительным сопротивлением к полюсам источника тока называется коротким замыканием , а максимальный ток для данного источника называется током короткого замыкания:

\ (~ I_ (kz) = \ frac (\ varepsilon) (r). \)

Для источников с малым значением r (например, для свинцово-кислотных аккумуляторов r = 0,1 — 0,01 Ом) ток короткого замыкания очень велик. Особенно опасно короткое замыкание в осветительных сетях от подстанций ( ε > 100 В), I кГц может достигать тысяч ампер.Во избежание возгораний в такие цепи включают предохранители.

Запишем закон Ома для замкнутой цепи при последовательном и параллельном подключении источников тока к батарее. При последовательном подключении источников «-» одного источника соединяется с «+» второго, «-» второго — с «+» третьего и так далее. (Рис.3, а). Если ε 1 = ε 2 = ε 3 a r 1 = r 2 = r 3, затем ε б = 3 ε 1, r b = 3 r один.В этом случае закон Ома для полной схемы равен \ [~ I = \ frac (\ varepsilon_b) (R + r_b) = \ frac (3 \ varepsilon_1) (R + 3r_1) \], или для n идентичных источников \ (~ I = \ frac (n \ varepsilon_1) (R + nr_1) \).

Последовательное соединение используется, когда внешнее сопротивление \ (~ R \ gg nr_1 \), затем \ (~ I = \ frac (n \ varepsilon_1) (R) \) и батарея может дать n в раз больше, чем ток от одного источника.

При параллельном подключении источников тока все источники «+» соединяются вместе, а источники «-» также вместе (рис.3, б). В данном случае

\ (~ \ varepsilon_b = \ varepsilon_1; \ r_b = \ frac (r_1) (3). \)

Откуда \ (~ I = \ frac (\ varepsilon_1) (R + \ frac (r_1) (3)) \).

Для n идентичных источников \ (~ I = \ frac (\ varepsilon_1) (R + \ frac (r_1) (n)) \).

Параллельное соединение источников тока используется, когда необходимо получить источник тока с низким внутренним сопротивлением или когда ток должен течь в цепи для нормальной работы потребителя электроэнергии.больше допустимого тока одного источника.

Параллельное соединение выгодно, когда R small по сравнению с R .

Иногда используется смешанное сочетание источников.

Литература

Аксенович Л.А. Физика в вузе: Теория. Задания. Тесты: Учебник. пособие для учреждений, обеспечивающих получение обсл. среда, образование / Л. А. Аксенович, Н.

alexxlab

Добавить комментарий

Ваш адрес email не будет опубликован. Обязательные поля помечены *